You are on page 1of 43

TEST - 1122 (EXPLANATION & SOURCE)

ALL INDIA SPECIAL TEST SERIES


TEST – 1122 (EXPLANATION & SOURCE)

1. Consider the following statements 2. Which of the statements are correct


regarding the Portuguese in India: regarding the later Mughals?
1. Vasco-da-Gama arrived at Calicut 1. Nadir Shah, the Persian emperor,
and was received by the local king, attacked India during the reign of
Zamorin. Muhammad Shah.
2. Portuguese established the first 2. Ahmad Shah Abdali was elected
fort at Cochin. the successor of Nadir Shah.
3. Cochin was declared as the capital 3. The Third Battle of Panipat in
of Portuguese. 1761 took place between the
Which of the statements given above Marathas and Ahmad Shah
are correct? Abdali.
(a) 1 and 2 only Select the correct answer using the
(b) 2 and 3 only code given below:
(c) 1 and 3 only (a) 1 and 2 only
(d) 1, 2 and 3 (b) 2 and 3 only
Explanation (c) 1 and 3 only
Statements 1 and 2 are correct (d) 1, 2 and 3
1498: Arrival of Vasco-da-Gama at Calicut Explanation
and his grand reception by the local king, Statements 1 and 2 are correct
Zamorin. Nadir Shah, the Persian emperor, attacked
1503: Establishment of the first Portuguese India in 1738-39, conquered Lahore and
fort at Cochin. defeated the Mughal army at Karnal on
1505: Establishment of the second February 13, 1739. Later, Muhammad Shah
Portuguese fort at Cannanore. was captured, and Delhi looted and
devastated. According to an estimate, apart
1509: Defeat of the combined fleet of
from the Peacock Throne and the Kohinoor
Gujarat, Egypt and Zamorin by the
diamond, seventy crore rupees were
Portuguese governor Francisco Almeida.
collected from the official treasury and the
1510: Alfonso Albuquerque, the Portuguese safes of the rich nobles.
governor, captures Goa from Bijapur.
Ahmad Shah Abdali (or Ahmad Shah
Statement 3 is incorrect Durrani), who was elected the successor of
1530: Declaration of Goa as the Portuguese Nadir Shah after the latter’s death in 1747,
capital. invaded India several times between 1748
1535: Subjugation of Diu. and 1767. He continuously harassed the
1559: The Portuguese capture Daman. Mughals who tried to buy peace in 1751-52
1596: Ouster of the Portuguese by the Dutch by ceding Punjab to him. In 1757, Abdali
from South-east Asia. captured Delhi and left behind an Afghan
caretaker to watch over the Mughal emperor.
1612: Loss of Surat to the English.
Before his return, Abdali had recognised
1663: The Dutch win all Portuguese forts on Alamgir II as the Mughal emperor and the
the Malabar coast to oust the Portuguese Rohilla chief, Najib-ud-Daula, as Mir
from India. Bakhshi of the empire, who was to act as
Source: Advent of the Europeans in India, personal ‘supreme agent’ of Abdali.
page No. 37 Statement 3 is correct:
1

● The first Battle of Panipat in 1526 was


Page

between Babur and Ibrahim Lodi. The


DELHI: VIJAY NAGAR 9717380832 & OLDRAJENDER NAGAR 9811293743 | JAIPUR: 8290800441
BENGALURU: KORMANGALA 7619166663 & CHANDRA LAYOUT 7619136663 | BHOPAL: 7509975361
PATNA: 7463950774 | INDORE: 7314977441 | RANCHI: 7463950774 | www.ksgindia.com
TEST - 1122 (EXPLANATION & SOURCE)
result of the battle laid the foundation Due to the conspiracy of the nawab’s
of the Mughal Empire by ending the officials, the 50,000-strong force of Siraj was
rule of the Delhi Sultanate. defeated by a handful of Clive’s forces. Siraj-
● The Second Battle of Panipat in 1556 ud-daula was captured and murdered by the
was between Akbar and Hemu; it order of Mir Jafar’s son, Miran.As a result of
decided in favour of the continuation this victory, Mir Jafar became the Nawab of
of the Mughal rule. Bengal. He gave large sums of money plus
● The Third Battle of Panipat in 1761 the zamindari of 24 parganas to the English.
between the Marathas and Ahmad Source: Expansion and Consolidation of
Shah Abdali put an end to the Maratha British Power, page No. 96
ambition of ruling over India.
Source: Advent of the Europeans in India, 4. Consider the following statements
page No. 65 regarding the Treaty of Allahabad:
1. Robert Clive concluded two
3. Consider the following statements treaties at Allahabad, one with the
regarding the Battle of Plassey: Nawab of Awadh and the other
1. The combined armies of Mir with the Mughal Emperor, Shah
Kasim, the Nawab of Awadh and Alam II.
Shah Alam II were defeated by the 2. Shah Alam II agreed to issue a
English forces under the battle. farman granting the diwani of
2. After the battle Mir Jafar became Bengal, Bihar and Orissa to the
the Nawab of Bengal. East India Company.
Which of the statements given above Which of the statements given above
is/are correct? are correct?
(a) 1 only (a) 1 only
(b) 2 only (b) 2 only
(c) Both 1 and 2 (c) Both 1 and 2
(d) Neither 1 nor 2 (d) Neither 1 nor 2
Explanation Explanation
Statement 1 is incorrect Both statements are correct
In Battle of Buxar, the combined armies of Robert Clive concluded two important
Mir Kasim, the Nawab of Awadh and Shah treaties at Allahabad in August 1765—one
Alam II were defeated by the English forces with the Nawab of Awadh and the other with
under Major Hector Munro at Buxar on the Mughal Emperor, Shah Alam II.
October 22, 1764 in a closely contested Nawab Shuja-ud-Daula agreed to:
battle. The English campaign against Mir (i) surrender Allahabad and Kara to
Kasim was short but decisive. Battle of Emperor Shah Alam II;
Plassey took place between Siraj-ud-daula (ii) pay Rs 50 lakh to the Company as war
and britishers. indemnity; and
Statement 2 is correct (iii) give Balwant Singh, Zamindar of
Clive forged a secret alliance with the traitors Banaras, full possession of his estate.
of the nawab—Mir Jafar, Rai Durlabh, Jagat Shah Alam II agreed to:
Seth (an influential banker of Bengal) and (i) reside at Allahabad, to be ceded to him
Omichand. Under the deal, Mir Jafar was to by the Nawab of Awadh, under the
be made the nawab who in turn would Company’s protection;
reward the Company for its services. The
(ii) issue a farman granting the diwani of
secret alliance of the Company with the
Bengal, Bihar and Orissa to the East
conspirators further strengthened the
India Company in lieu of an annual
English position. So the English victory in
payment of Rs 26 lakh; and
2

the Battle of Plassey (June 23, 1757) was


decided before the battle was even fought.
Page

DELHI: VIJAY NAGAR 9717380832 & OLDRAJENDER NAGAR 9811293743 | JAIPUR: 8290800441
BENGALURU: KORMANGALA 7619166663 & CHANDRA LAYOUT 7619136663 | BHOPAL: 7509975361
PATNA: 7463950774 | INDORE: 7314977441 | RANCHI: 7463950774 | www.ksgindia.com
TEST - 1122 (EXPLANATION & SOURCE)
(iii) a provision of Rs 53 lakh to the life, the turn of the seasons, the highlights of
Company in return for nizamat the agricultural calendar, religious festivals
functions (military defence, police, and and important events that punctuate the
administration of justice) of the said flow of life, such as births and marriages.
provinces. While folk music and dance share common
Source: Expansion and Consolidation of themes and concerns, there is a wide variety
British Power, page No. 100 of forms. Along the entire Himalayan region,
from Kashmir to Darjeeling, folk dancers link
arms and sway gracefully in undulating
5. Which one of the following statements
movements, celebrate the sowing of the
about women of Rig Veda Society is
wheat crop; few can resist the infectious beat
not correct?
of the dholak, the two-sided drum, and pairs
(a) They participated in all the major of dancers take turns to execute complex
ceremonies. acrobatic movements in the centre of a circle
(b) They were given equal of abandoned dancers.
opportunities as men in Women perform the Giddha, also
intellectual development. characterised by its spontaneous energy.
(c) There were no prominent Rajasthani women, their faces covered with
women poets during the Rig flowing veils, are swirls of colour as they
Vedic period. pirouette in the Ghoomar dance, while their
(d) There was no child marriage and counterparts in Gujarat perform the famous
the practice of sati was absent Garba, dancing in a circle with batons. Their
Explanation: men perform the Dandiya Ras, a more
Option (c) is the correct answer:The wife vigorous version of the same dance, leaping
took care of the household and participated and crouching in twirling patterns. In the
in all the major ceremonies. Women were fishing communities of Maharashtra, men
given equal opportunities as men for their and women link arms and dance together
spiritual and intellectual development. and the women climb on to the men's
There were women poets like Apala, shoulders to form pyramids. The women's
Viswavara, Ghosa and Lopamudra during Lavani dance from this area is notable for its
the Rig Vedic period. Women could even unabashed sensuality. There are also several
attend the popular assemblies. There was no forms of dance-drama or folk theatre, such
child marriage and the practice of sati was as the Nautanki of Rajasthan, Uttar Pradesh
absent. and Bihar, the Bhavai of Gujarat, the
irreverent Tamasha of Maharashtra,the
Source:Class11 History/TN Board/Pg. 29
Bengali Jatra, the spectacular Yakshagana
of Karnataka and Theyyam of Kerala, all of
6. Which of the following pair(s) is/are which narrate legends of local heroes, kings
correctly matched? and deities. Martial art forms throughout the
Performative Art State country have been stylized to quasi dance
1. Bhavai Maharashtra forms, notable among which are the martial
2. Jatra Bengal dances of the North-eastern hill tribes, the
Lazim dances of Maharashtra, the
3. Lazim Tamil Nadu
Kalaripayattu of Kerala, and the highly
Select the correct answer using the stylized masked Chhau dances of Orissa,
code given below: West Bengal and Bihar.
(a) 1 and 3 only Source:http://ccrtindia.gov.in/performinga
(b) 2 and 3 only rt.php
(c) 2 only
(d) 1, 2 and 3
Explanation:Option (c) is correct
3

The folk music and dances of agricultural


communities celebrate the rhythms of daily
Page

DELHI: VIJAY NAGAR 9717380832 & OLDRAJENDER NAGAR 9811293743 | JAIPUR: 8290800441
BENGALURU: KORMANGALA 7619166663 & CHANDRA LAYOUT 7619136663 | BHOPAL: 7509975361
PATNA: 7463950774 | INDORE: 7314977441 | RANCHI: 7463950774 | www.ksgindia.com
TEST - 1122 (EXPLANATION & SOURCE)
7. Consider the following pairs: on liberal principles. It described itself as the
Social Major standard-bearer of Mohammedan
Reformers reforms Renaissance, and based itself, like the
Brahmo Samaj, on the principles of
1. Haji Eradication universal religion of all humanity, opposing
Shariatullah of social jihad (sacred war against non-Muslims). The
innovations movement spread Western liberal education
or un- among the Indian Muslims.
Islamic
Source:A Brief History of Modern India, Page
practices
249, 252
2. E.V. Rejection of
Ramaswamy the
8. Which one of the following is not
Naicker brahminical
correct about the August Declaration
religion and
of 1917?
culture
(a) The Secretary of State for India,
3. Mirza Spread John Morley made this
Ghulam Western statement in August 1917.
Ahmad liberal
(b) It talked about an increasing
education
participation of Indians in every
among the
branch of administration
Indian
Muslims (c) It aimed to provide responsible
government in India as an integral
part of the British Empire.
Which of the pairs given above is/are (d) The demand by nationalists for
correctly matched? home rule could not be termed as
(a) 1 and 2 only seditious after this statement.
(b) 3 only Explanation:
(c) 2 and 3 only Option (a) is correct answer:
(d) 1, 2 and 3 ·The Secretary of State for India, Edwin
Explanation: Samuel Montagu, made a statement on
Option (d) is correct: August 20, 1917 in the British House of
Faraizi movement, also called the Fara’idi Commons in what has come to be known
Movement because of its emphasis on the as the August Declaration of 1917.
Islamic pillars of faith, was founded by Haji ·The statement said: “The government policy
Shariatullah in 1818. Its scene of action was is of an increasing participation of Indians in
East Bengal, and it aimed at the eradication every branch of administration and gradual
of social innovations or un-Islamic practices development of self-governing institutions
current among the Muslims of the region with a view to the progressive realisation of
and draw their attention to their duties as responsible government in India as an
Muslims. integral part of the British Empire.”
Self-Respect Movement movement was From now onwards, the demand by
started by E.V. Ramaswamy Naicker, a nationalists for self government or home rule
Balija Naidu, in the mid-1920s. The could not be termed as seditious since
movement aimed at nothing short of a attainment of self-government for Indians
rejection of the brahminical religion and now became a government policy, unlike
culture which Naicker felt was the prime Morley’s statement in 1909 that the reforms
instrument of exploitation of the lower were not intended to give self-government to
castes. India.
The Ahmadiyya forms a sect of Islam which Source: Modern History/ Spectrum/ pg.
originated from India. It was founded by 337
4

Mirza Ghulam Ahmad in 1889. It was based


Page

DELHI: VIJAY NAGAR 9717380832 & OLDRAJENDER NAGAR 9811293743 | JAIPUR: 8290800441
BENGALURU: KORMANGALA 7619166663 & CHANDRA LAYOUT 7619136663 | BHOPAL: 7509975361
PATNA: 7463950774 | INDORE: 7314977441 | RANCHI: 7463950774 | www.ksgindia.com
TEST - 1122 (EXPLANATION & SOURCE)
9. Which of the following statements are 1667: Aurangzeb gave the English a farman
correct regarding British advent in for trade in Bengal.
India? 1691: The Company got the imperial order to
1. William Hawkins arrived at continue their trade in Bengal in lieu of
Aurangzeb’s court. payment of Rs 3,000 a year.
2. The Company established its first 1717: The Mughal emperor Farrukhsiyar
factory in the south in issued a farman, called Magna Carta of the
Masulipatnam. Company, giving the Company a large
3. Farrukhsiyar issued a farman, number of trade concessions.
called Magna Carta of the Source: Advent of the Europeans in India,
Company, giving the Company a page No. 45
large number of trade
concessions. 10. Consider the following statements
Select the correct answer using the about puppetry in India:
codes given below: 1. The art of puppetry finds mention
(a) 1 and 2 only in Tamil classic
(b) 2 and 3 only ‘Silappadikaaram’.
(c) 1 and 3 only 2. Gombeyatta is a string puppet
(d) 1, 2 and 3 form from Karnataka.
Explanation: option b is. Correct: Which of the Statements given above
1600: The East India Company was is/are correct?
established. (a) 1 only
1609: William Hawkins arrived at Jahangir’s (b) 2 only
court. (c) Both 1 and 2
1611: Captain Middleton obtained the (d) Neither 1 nor 2
permission of the Mughal governor of Surat Explanation:
to trade there. Statement 1 is correct
1613: A permanent factory of East India The earliest reference to the art of puppetry
Company was established at Surat. is found in Tamil classic
1615: Sir Thomas Roe, the ambassador of ‘Silappadikaaram’ written around the 1st
King James I, arrived at Jahangir’s court. By or 2nd century B.C.
1618, the ambassador succeeded in Natyashastra, the masterly treatise on
obtaining two farmans (one each from the dramaturgy written sometime during 2nd
emperor and Prince Khurram) confirming century BC to 2nd century AD., does not
free trade with exemption from inland tolls. refer to the art of puppetry but the producer-
1616: The Company established its first cum-director of the human theatre has been
factory in the south in Masulipatnam. termed as ‘Sutradhar’ meaning the holder of
1632: The Company got the golden farman strings.
from the Sultan of Golconda which ensured The word might have found its place in
safety and prosperity of their trade. theatre-terminology long before
1633: The Company established its first Natyashastra was written but it must come
factory in east India in Hariharpur, Balasore from marionette theatre.
(Odisha). Puppetry, therefore, must have originated in
1639: The Company got the lease of Madras India more than 500 years before Christ.
from a local king. Statement 2 is correct
1651: The Company was given permission to
trade at Hooghly (Bengal). String Puppet form State
1662: The British King, Charles II, was given
Bombay as dowry for marrying a Portuguese Kathputli Rajasthan
5

princess (Catherine of Braganza).


Page

DELHI: VIJAY NAGAR 9717380832 & OLDRAJENDER NAGAR 9811293743 | JAIPUR: 8290800441
BENGALURU: KORMANGALA 7619166663 & CHANDRA LAYOUT 7619136663 | BHOPAL: 7509975361
PATNA: 7463950774 | INDORE: 7314977441 | RANCHI: 7463950774 | www.ksgindia.com
TEST - 1122 (EXPLANATION & SOURCE)
➢ Asoka convened the Third Buddhist
Kundhei Orissa Council at Pataliputra in 240 B.C. in
order to strengthen the Sangha. It was
Gombeyatta Karnataka presided over by MoggaliputtaTissa.
Source:Class11 History/TN Board/Pg. 64
Bommalattam (string Tamil Nadu
and Rod fusion) 12. Consider the following statements with
reference to a performative art:
Source: 1. Bharata Muni's Natya Shastra
http://ccrtindia.gov.in/puppetforms.php provides information on this
dance form.
11. With reference to Ashoka and 2. We find visual evidence of this
Buddhism, consider the following dance on the gopurams of the
statements: Chidambaram temple.
1. He appointed special officers 3. The style was kept alive by the
called Dharma Mahamatras to devadasis.
speed up the progress of The above statements refer to which of
Dhamma. the following dances?
2. He sent a mission to Sri Lanka (a) Mohiniyattam
under his son Mahendra and (b) Kuchipudi
daughter Sangamitra. (c) Odissi
3. He convened the Third Buddhist (d) Bharatnatyam
Council at Pataliputra in 240 B.C. Explanation: Option (d) is correct
Which of the statements given above Bharatnatyam Dance is considered to be
are correct? over 2000 years old. Several texts beginning
(a) 1 and 2 only with Bharata Muni's Natya Shastra (200
(b) 1 and 3 only B.C.E. to 200 C.E.) provide information on
(c) 2 and 3 only this dance form. The Abhinaya Darpana by
(d) 1, 2 and 3 Nandikesvara is one of the main sources of
textual material, for the study of the
Explanation:
technique and grammar of body movement
All statements are correct: According to in Bharatnatyam Dance. There is also a
some scholars, his conversion to Buddhism great deal of visual evidence of this dance
was gradual and not immediate. About 261 form in paintings and stone and metal
B.C. Asoka became a Sakya Upasaka (lay sculptures of ancient times. On the
dsicple) and two and a half years later, a gopurams of the Chidambaram temple, one
Bikshu (monk). Then he gave up hunting, can see a series of Bharatnatyam poses,
visited Bodh-Gaya, and organized missions. frozen in stone as it were, by the sculptor. In
➢ He appointed special officers called many other temples, the charis and karanas
Dharma Mahamatras to speed up the of the dance are represented in sculpture
progress of Dhamma. and one can make a study of the dance form.
➢ In 241 B.C., he visited the birth place Bharatnatyam dance is known to be
of Buddha, the Lumbini Garden, near ekaharya, where one dancer takes on many
Kapilavastu. roles in a single performance. In the early
➢ He also visited other holy places of 19th century, the famous Tanjore Quartette,
Buddhism like Sarnath, Sravasti and under the patronage of Raja Serfoji are said
Kusinagara. to have been responsible for the repertoire of
➢ He sent a mission to Sri Lanka under Bharatnatyam dance as we see it today.
his son Mahendra and daughter The style was kept alive by the devadasis,
Sangamitra who planted there the who were young girls 'gifted' by their parents
6

branch of the original Bodhi tree. to the temples and who were married to the
Page

DELHI: VIJAY NAGAR 9717380832 & OLDRAJENDER NAGAR 9811293743 | JAIPUR: 8290800441
BENGALURU: KORMANGALA 7619166663 & CHANDRA LAYOUT 7619136663 | BHOPAL: 7509975361
PATNA: 7463950774 | INDORE: 7314977441 | RANCHI: 7463950774 | www.ksgindia.com
TEST - 1122 (EXPLANATION & SOURCE)
gods. The devadasis performed music and attended by 72 delegates and presided over
dance as offerings to the deities, in the by Womesh Chandra Bonnerjee. Hereafter,
temple courtyards. Some of the renowned the Congress met every year in December, in
performers and gurus of the early part of the a different part of the country each time.
century belong to the devadasi families, a Some of the great presidents of the Congress
well-known name is BalaSaraswati. during this early phase were Dadabhai
Source:http://ccrtindia.gov.in/bharatnatya Naoroji (thrice president), Badruddin Tyabji,
m.php Pherozshah Mehta, P. Anandacharlu,
Surendranath Banerjea, Romesh Chandra
Dutt, Ananda Mohan Bose and Gopal
13. With reference to Rig Vedic Society,
Krishna Gokhale.
consider the following statements:
Statement 2 is incorrect: In 1890,
1. The head of the family was known
Kadambini Ganguly, the first woman
as grahapathi.
graduate of Calcutta University, addressed
2. Polygamy was not allowed in Rig the Congress session, which symbolised the
Vedic society. commitment of the freedom struggle to give
Which of the statements given above the women of India their due status in
is/are correct? national life. She was also the first
(a) 1 only female speaker at the Indian National
(b) 2 only Congress session.
(c) Both 1 and 2 Source: Modern History/ Spectrum/ pg.
(d) Neither 1 nor 2 274-275
Explanation: https://www.indiatoday.in/education-
today/gk-current-affairs/story/kadambini-
Statement 1 is correct: The Rig Vedic
ganguli-india-s-first-female-doctor-who-
society was patriarchal. The basic unit of
made-calcutta-medical-college-start-
society was family or graham. The head of
admitting-women-1570858-2019-07-18
the family was known as grahapathi.
Statement 2 is incorrect: Monogamy was
generally practiced while polygamy was 15. Consider the following statements:
prevalent among the royal and noble 1. William Bentinck established the
families. The wife took care of the household first medical college in India to
and participated in all the major ceremonies. give instructions in modern
Source:Class11 History/TN Board/ pg. 29 medicine.
2. Lord Curzon laid the foundation of
Pusa Agriculture College.
14. With reference to Indian National
Congress (INC), consider the following Which of the statements given above
statements: is/are correct?
1. Dadabhai Naoroji presided over (a) 1 only
the first session of the INC. (b) 2 only
2. Annie Besant was the first woman (c) Both 1 and 2
to address a session of the INC. (d) Neither 1 nor 2
Which of the statements given above Explanation:
is/are correct? Statement 1 is correct: Medical training
(a) 1 only started with establishment of a medical
(b) 2 only college in Calcutta in 1835 during the reign
(c) Both 1 and 2 of William Bentinck.
(d) Neither 1 nor 2 Statement 2 is correct: Lord Curzon
established an Agriculture College at Pusa
Explanation:
which acted as a parent institution of similar
Statement 1 is incorrect: The first session institutions in other provinces.
7

of the Indian National Congress was


Page

DELHI: VIJAY NAGAR 9717380832 & OLDRAJENDER NAGAR 9811293743 | JAIPUR: 8290800441
BENGALURU: KORMANGALA 7619166663 & CHANDRA LAYOUT 7619136663 | BHOPAL: 7509975361
PATNA: 7463950774 | INDORE: 7314977441 | RANCHI: 7463950774 | www.ksgindia.com
TEST - 1122 (EXPLANATION & SOURCE)
Source: A Brief History of India; Chapter: buildings with a roof that rises into a vaulted
Chapter 30: Development of Education; Page chamber.
642 Source:Ch-6, Temple Architecture and
Sculpture,An Introduction toIndian Art Part-
16. Which of the following are the I, Class-XI.
architectural forms of Nagara
Temples? 17. Consider the following statements:
1. Phamsana 1. As per ‘Treaty of Allahabad’,
2. Shikhara Nawab Shuja-ud-Daula agreed to
3. Valabhi surrender Allahabad and Kara to
Select the correct answer using the the British.
code given below: 2. As per ‘Treaty of Bassein’, Peshwa
(a) 1 and 2 only agreed to subject his relations
with other states to the control of
(b) 2 and 3 only
the English.
(c) 1 and 3 only
Which of the statements given above
(d) 1, 2 and 3 is/are correct?
Explanation: (a) 1 only (b) 2 only
Option (d) is correct: (c) Both 1 and 2 (d) Neither 1 nor 2
There are many subdivisions of nagara Explanation:
temples depending on the shape of the
Statement 1 is incorrect: According to
shikhara. There are different names for the
treaty of Allahabad, Nawab Shuja-ud-Daula
various parts of the temple in different parts
agreed to:
of India; however, the most common name
for the simple shikhara which is square at (i) surrender Allahabad and Kara to
the base and whose walls curve or slope Emperor Shah Alam II;
inward to a point on top is called the 'latina' (ii) pay Rs 50 lakh to the Company as war
or the rekha-prasada type of shikara. indemnity;and
The second major type of architectural form (iii) give Balwant Singh, Zamindar of
in the nagaraorder is the phamsana. Banaras, full possession of his estate.
Phamsana Buildings tend to be broader and Statement 2 is correct:Treaty of Bassein
shorter than latina ones. Their roofs are (1802) Under the treaty, the Peshwa Agreed:
composed of several slabs that gently rise to (i) to receive from the Company a native
a single point over the centre of the building, infantry(consisting of not less than
unlike the latina ones which look like 6,000 troops), with the usual
sharply rising tall towers. Phamsana roofs proportion of field artillery and
do not curve inward, instead they slope European artillery men attached, to be
upwards on a straight incline. In many North permanently stationed in his
Indian temples you will notice that the territories;
phamsana design is used for the mandapas (ii) to cede to the Company territories
while the main garbhagriha is housed in a yielding an income of Rs 26 lakh;
latina building. Later on, the latina buildings
(iii) to subject his relations with other
grew complex, and instead of appearing like
states to the control of the English.
a single tall tower, the temple began to
support many smaller towers, which were Source: A Brief History of Modern India,
clustered together like rising mountain- Page 100, 113, 136
peaks with the tallest one being in the
centre, and this was the one which was 18. With reference to Vaisheshika school,
always above the garbhagriha. consider the following statements:
The third main sub-type of the nagara 1. It gives importance to the
building is what is generally called the discussion of material elements or
8

valabhi type. These are rectangular dravya.


Page

DELHI: VIJAY NAGAR 9717380832 & OLDRAJENDER NAGAR 9811293743 | JAIPUR: 8290800441
BENGALURU: KORMANGALA 7619166663 & CHANDRA LAYOUT 7619136663 | BHOPAL: 7509975361
PATNA: 7463950774 | INDORE: 7314977441 | RANCHI: 7463950774 | www.ksgindia.com
TEST - 1122 (EXPLANATION & SOURCE)
2. It propounded the atom theory iii) Establish the headquarters for a
believing that all material objects movement;
are made up of atoms. iv) Promote friendly relations among
Which of the statements given above nationalist political workers from
is/are correct? different parts of the country;
(a) 1 only v) Develop and propagate an anti-
(b) 2 only colonial nationalist ideology;
(c) Both 1 and 2 vi) Formulate and present popular
(d) Neither 1 nor 2 demands before the government with a
view to unifying the people over a
Explanation:
common economic and political
Statement 1 and Statement 2 are correct- programme;
The Vaisheshika school gives importance to
vii) Develop and consolidate a feeling of
the discussion of material elements or
national unity among people
dravya. They draw a line between
irrespective of religion, caste or
particularities and their aggregate. Earth,
province.
water, fire, air, and ether (sky), when
combined, give rise to new objects. The viii) Carefully promote and nurture Indian
Vaisheshika school propounded the atom nationhood.
theory believing that all material objects are Social reform was not on the agenda of the
made up of atoms. The Vaisheshika thus INC in initial days.
marked the beginning of physics in India, Source: Modern History/ Spectrum/ pg.
but the scientific view was diluted by a belief 276
in god and spiritualism, and this school put
its faith in both heaven and salvation. 20. Consider the following statements
Source-India’s Ancient Past, R.S. Sharma, about ancient Indian history:
Chapter 29,Page-318 1. Vikramaditya issued the first-ever
long inscription in chaste
19. Which of the following was/were the Sanskrit.
aims and objectives of the Indian 2. Vikram Samvat started in 57 B.C,
National Congress in the initial stage? which is now used by the
1. Establish the headquarters for a Government of India.
movement. 3. The Shaka era started in A.D 78
2. Insure social reforms at rapid Select the correct answer using the
pace. code given below:
3. Develop and propagate an anti- (a) 2 only
colonial nationalist ideology (b) 1 and 3 only
Select the correct answer using the (c) 2 and 3 only
code given below:
(d) 3 only
(a) 1 only
Explanation:
(b) 1 and 3 only
Statement 1 is incorrect
(c) 2 and 3 only
The most famous Shaka ruler in India was
(d) 1, 2 and 3 Rudradaman I (AD 130-150). He is famous
Explanation: in history because of the repairs he
Option (b) is correct: The main aim of the undertook to improve the Sudarshana lake
Indian National Congress in the initial stage in the semi-arid zone of Kathiawar. He also
were to— was a great lover of Sanskrit. Although a
i) Found a democratic, nationalist foreigner settled in India, he issued the first
movement; ever long inscription in chaste Sanskrit.
ii) Politicise and politically educate All the earlier longer inscriptions that we
9

people;
Page

DELHI: VIJAY NAGAR 9717380832 & OLDRAJENDER NAGAR 9811293743 | JAIPUR: 8290800441
BENGALURU: KORMANGALA 7619166663 & CHANDRA LAYOUT 7619136663 | BHOPAL: 7509975361
PATNA: 7463950774 | INDORE: 7314977441 | RANCHI: 7463950774 | www.ksgindia.com
TEST - 1122 (EXPLANATION & SOURCE)
have in this country were composed in Nizam into his ally. He then joined the Nizam
Prakrit. to attack the Nawab of Arcot.
Statement 2 is incorrect The war continued for a year-and-a-half
In about 57-58 B.C we hear of king of Ujjain without any conclusion. Haidar changed his
who effectively fought against the Shakas strategy and suddenly appeared before the
and succeeded in driving them out in his gates of Madras. There was complete chaos
time. He called himself Vikramaditya, and an and panic at Madras forcing the English to
era called the Vikrama Samvat is reckoned conclude a very humiliating treaty with
from the event of his victory over the Shakas Haidar on April 4, 1769—Treaty of
in 57 B.C. However, this is NOT the official Madras. The treaty provided for the
calendar used by the Government of exchange of prisoners and mutual
India. restitution of conquests. Haidar Ali was
Statement 3 is correct promised the help of the English in case he
was attacked by any other power.
Kanishka was the most famous kushan
ruler. He started an era in A.D 78, which Treaty of Seringapatam was signed during
is now known as the Shaka era and is used third anglo-mysore war. Under this treaty
by the Government of India. He also of 1792, nearly half of the Mysorean territory
extended his wholehearted patronage to was taken over by the victors. Baramahal,
Buddhism. He held a Buddhist council in Dindigul and Malabar went to the English,
Kashmir, where the doctrines of the while the Marathas got the regions
Mahayana form of Buddhism were finalized. surrounding the Tungabhadra and its
He was also a great patron of art and tributaries and the Nizam acquired the areas
Sanskrit literature. from the Krishna to beyond the Pennar.
Besides, a war damage of three crore rupees
Source:
was also taken from Tipu. Half of the war
Ancient India (Ram Sharan Sharma, class indemnity was to be paid immediately while
XI), Chapter 16, Page 143, 144 the rest was to be given in installments, for
which Tipu’s two sons were taken as
21. Which of the following statements hostages by the English.
is/are correct regarding Anglo-Mysore Source: Expansion and Consolidation of
wars? British Power, page No. 105-106
1. Treaty of madras was signed after
second Anglo-Mysore war. 22. With reference to Samkhya school of
2. Treaty of Seringapatam was philosophy, consider the following
signed during the third Anglo- statements:
Mysore war. 1. According to the early Samkhya
Select the correct answer using the philosophy, the presence of divine
code given below: agency is not essential to the
(a) 1 only creation of the world.
(b) 2 only 2. A person can attain salvation
(c) Both 1 and 2 through the acquisition of real
(d) Neither 1 nor 2 knowledge.
Explanation: Which of the statements given above
is/are correct?
Statement 1 and 2 are incorrect:
(a) 1 only
During the first anglo-mysore war The
Nizam, the Marathas, and the English allied (b) 2 only
together against Haidar Ali. Haidar acted (c) Both 1 and 2
with considerable tact and diplomatic skill. (d) Neither 1 nor 2
He paid the Marathas to turn them neutral Explanation:
10

and, promising to share conquered Statement 1 is correct and Statement 2 is


territories with the Nizam, converted the incorrect-Samkhya, literally ‘count’, seems
Page

DELHI: VIJAY NAGAR 9717380832 & OLDRAJENDER NAGAR 9811293743 | JAIPUR: 8290800441
BENGALURU: KORMANGALA 7619166663 & CHANDRA LAYOUT 7619136663 | BHOPAL: 7509975361
PATNA: 7463950774 | INDORE: 7314977441 | RANCHI: 7463950774 | www.ksgindia.com
TEST - 1122 (EXPLANATION & SOURCE)
to have originated first. According to the (a) 1 only
early Samkhya philosophy, the presence of (b) 2 only
divine agency is not essential to the creation (c) Both 1 and 2
of the world. The world owes its creation and
(d) Neither 1 nor 2
evolution more to Nature or prakriti than to
God. This was a rational and scientific view. Explanation:
Around the fourth century AD, in addition to Statement 1 is incorrect: The home of the
prakriti, purusha or spirit was introduced as Gandhara school of art is the territory in and
an element in the Samkhya system, and the around Peshawar in northwestern India. The
creation of the world was attributed to both. best of the Gandhara sculpture was
According to the new view, Nature and the produced during the first and second
spiritual element together create the world. centuries A.D. It originated during the reign
Thus, at the outset the Samkhya school of of Indo-Greek rulers but the real patrons of
philosophy was materialistic, but later it this school of art were the Sakas and the
tended to become spiritualistic. Initially, Kushanas, particularly Kanishka. Gandhara
according to this school, a person can attain art was a blend of Indian and Graeco-
salvation through the acquisition of real Roman elements.
knowledge, and his misery can be ended for Statement 2 is correct: Specimens of
ever. This knowledge can be acquired Gandhara sculpture have been found in
through perception (pratyaksha), inference Taxila, Peshawar and in several places of
(anumana), and hearing (shabda). Such a northwest India. The Gandhara school made
method is characteristic of a scientific sculptures of the Buddha in various sizes,
system of inquiry. shapes and postures. The reliefs depict
According to the Yoga school, a person can Buddha’s birth, his renunciation and his
attain salvation through meditation and preaching. The salient features of Gandhara
physical application. Practice of control over art are:
pleasure, the senses, and bodily organs is ➢ Moulding human body in a realistic
central to this system. In order to obtain manner with minute attention to
salvation, physical exercises in various physical features like muscles,
postures called asanas are prescribed, and a moustache and curtly hair.
breathing exercise called pranayama is ➢ Thick drapery with large and bold fold
recommended. It is thought that through lines.
these methods, the mind gets diverted from ➢ Rich carving, elaborate ornamentation
worldly matters and achieves concentration. and symbolic expressions.
These exercises are important because they
➢ The main theme was the new form of
not only presuppose some development of
Buddhism – Mahayanism and the
the knowledge of physiology and anatomy in
evolution of an image of Buddha.
ancient times, but they also indicate a
tendency to run away from worldly Source: Class11 History/TN Board/Pg. 80-
difficulties. 81
Source-India’s Ancient Past, R.S. Sharma,
Chp 29, Page-317 24. Consider the following:
1. Bhitargaon in Kanpur
23. With reference to Gandhara School of 2. Bhitari in Ghazipur
art, consider the following statements: 3. Deogarh in Jhansi
1. This school of art developed on Which of the above brick temples were
indigenous lines only. built during the Gupta period?
2. Its reliefs depict Buddha’s birth, (a) 1 and 2 only
his renunciation and his (b) 1 and 3 only
preaching. (c) 2 and 3 only
11

Which of the statements given above (d) 1, 2 and 3


is/are correct?
Explanation: Option (d) is correct.
Page

DELHI: VIJAY NAGAR 9717380832 & OLDRAJENDER NAGAR 9811293743 | JAIPUR: 8290800441
BENGALURU: KORMANGALA 7619166663 & CHANDRA LAYOUT 7619136663 | BHOPAL: 7509975361
PATNA: 7463950774 | INDORE: 7314977441 | RANCHI: 7463950774 | www.ksgindia.com
TEST - 1122 (EXPLANATION & SOURCE)
As the Guptas supported Brahmanism, service without the prior approval of the
images of Vishnu, Shiva, and some other British. Nor could he negotiate with any
Hindu gods were fashioned for the first time other Indian ruler without consulting the
during their period. At many places, the governor-general. In return for all this, the
entire pantheon is portrayed with the chief British would defend the ruler from his
god at the centre surrounded by his enemies and adopt a policy of
retainers and subordinates. The leading god noninterference in the internal matters of
is represented as large in size, with his the allied state.
retainers and subordinate gods drawn on a Source: Expansion and Consolidation of
smaller scale. This reflects clear social British Power, page No. 131
hierarchy and discrimination. The Gupta
period was poor in terms of architecture. All
26. Which among the following is/are the
that we find are a few temples made of brick
principal ports of the Bahmani
in UP and a stone temple. The brick temples
Kingdom?
of Bhitargaon in Kanpur, Bhitari in
Ghazipur, and Deogarh in Jhansi may be 1. Dabhol
mentioned. The Buddhist university at 2. Chaul
Nalanda was set up in the fifth century, and 3. Krishnapatnam
its earliest structure, made of brick, relates 4. Malabar
to this period. Select the correct answer using the
Source-India’s Ancient Past, R.S. Sharma, code given below:
Chapter-27,Page 287 (a) 2 and 3 only
(b) 1 and 2 only
25. Consider the following statements (c) 1 and 3 only
regarding the Subsidiary alliance:
(d) 1, 2, 3 and 4
1. The subsidiary alliance system
Explanation: Option (b) is correct.
was proposed by Lord Wellesley,
who was the governor-general. Firuz Bahmani started the Bahmani
expansion towards Berar by defeating the
2. Under this system, the Indian
Gond raja, Narsing Rai of Kherla. The rai
ruler could not employ any
made a present of 40 elephants, 5 maunds
European in his service without
of gold and 50 maunds of silver. A daughter
the prior approval of the British.
of the rai was also married to Firuz. Kherla
Which of the statements given above was restored to Narsing who was made an
is/are correct? amir of the kingdom and given robes of state,
(a) 1 only including an embroidered cap.Firuz Shah
(b) 2 only Bahmani paid much attention to the
(c) Both 1 and 2 principal ports of his kingdom, Chaul and
(d) Neither 1 nor 2 Dabhol, which attracted trading ships from
the Persian Gulf and the Red Sea, and
Explanation
brought in luxury goods from all parts of the
Both statements are correct world.
The subsidiary alliance system was used by Source-History of Medieval India, Satish
Lord Wellesley, who was governor-general Chandra, Chapter-9, Page-163
from 1798-1805, to build an empire in India.
Under the system, the allying Indian state’s
ruler was compelled to accept the permanent 27. Which among the following
stationing of a British force within his events/reforms is Ishwar Chandra
territory and to pay a subsidy for its Vidyasagar associated with?
maintenance. Also, the Indian ruler had to 1. Legalizing widow remarriage.
agree to the posting of a British resident in 2. Opposing polygamy.
12

his court. Under the system, the Indian ruler 3. Advocating higher education of
could not employ any European in his women.
Page

DELHI: VIJAY NAGAR 9717380832 & OLDRAJENDER NAGAR 9811293743 | JAIPUR: 8290800441
BENGALURU: KORMANGALA 7619166663 & CHANDRA LAYOUT 7619136663 | BHOPAL: 7509975361
PATNA: 7463950774 | INDORE: 7314977441 | RANCHI: 7463950774 | www.ksgindia.com
TEST - 1122 (EXPLANATION & SOURCE)
4. Right of women to abortion
5. Opposing child marriage. 29. Which of the following temples are
6. Break the priestly monopoly of located at Aihole in Karnataka?
scriptural knowledge 1. The Ladkhan temple
Select the correct answer using the 2. The Raja Rani temple
code given below. 3. The Durga temple
(a) 1,3, 5 and 6 only Select the correct answer using the
(b) 1,2 and 4 only code given below:
(c) 2, 3 and 4 only (a) 1 and 3 only
(d) 1, 2, 3, 5 and 6 only (b) 2 and 3 only
Explanation: (c) 1 only
Option (d) is correct: (d) 1, 2 and 3
In 1850, he became the principal of Sanskrit Explanation: Option (a) is correct
College. He was determined to break the The Ladkhan temple of Aihole belongs to
priestly monopoly of scriptural knowledge, about 5th century A.D. Here the architect
and for this he opened the Sanskrit College has tried to give attention to the
to non-brahmins. circumambulation path which is enclosed by
Vidyasagar started a movement in support of means of a wall allowing devotees to have
widow remarriage which resulted in pradakshina or circumambulatory of the
legalization of widow remarriage. He was holy of holies.
also a crusader against child marriage and The Durga temple at Aihole is an apsidal
polygamy. He did much for the cause of temple of about 550 A.D. in which the
women’s education. architect has made immense improvements
Right of women to abortion was not among upon his previous attempts. This temple is
the causes advocated by Vidyasagar. provided with a high pedestal, an open
Source:A Brief History of Modern India, Page pillared verandah serving as
236 pradakshanapatha, in place of a dark,
ambulatory passage as in the case of the
Ladkhan temple.
28. The Lion Capital at Sarnath was built
in commemoration of which of the The Raja Rani Temple of Bhubaneswar, is
following historical event? superbly decorated, showing sensuous and
graceful figures of Yakshis and Vrikshikas
(a) Four noble truth of Buddha
standing amidst luxurious natural
(b) Eight fold path of Buddha surroundings.
(c) First sermon of Buddha Source:
(d) Enlightenment of Buddha http://ccrtindia.gov.in/templearchitecture.
Explanation: php
Option (c) is correct:
The Lion Capital discovered more than a 30. Consider the following statements
hundred years agoat Sarnath, near about the age of Satavahanas in
Varanasi, is generally referred to as Deccan:
SarnathLion Capital. This is one of the finest 1. Satvahanas issued coins of gold
examples of sculpture from the Mauryan and lead.
period. Built in commemoration of the
2. The people of Deccan knew the art
historical event of the first sermon or the
of paddy transplantation.
Dhammachakra Pravartan by the Buddha at
Sarnath, the capital was built by Ashoka. 3. Prakrit was the official language of
the Satavahanas.
Source: Ch-3, Arts of Maurya Period, An
Which of the Statements given above
13

Introduction toIndian Art Part-I, Class-XI.


is/are correct?
(a) 1 and 2 only
Page

DELHI: VIJAY NAGAR 9717380832 & OLDRAJENDER NAGAR 9811293743 | JAIPUR: 8290800441
BENGALURU: KORMANGALA 7619166663 & CHANDRA LAYOUT 7619136663 | BHOPAL: 7509975361
PATNA: 7463950774 | INDORE: 7314977441 | RANCHI: 7463950774 | www.ksgindia.com
TEST - 1122 (EXPLANATION & SOURCE)
(b) 1 and 3 only The Bombay Presidency Association was
(c) 1, 2 and 3 started by Badruddin Tyabji, Pherozshah
(d) 2 and 3 only Mehta and K.T. Telang in 1885.
Explanation: Political Associations in Madras
Statement 1 is incorrect The Madras Mahajan Sabha was founded in
1884 by M. Viraraghavachari, B.
The Satavahanas may have exploited the
Subramaniya Aiyer and P. Anandacharlu.
iron ores of Karimnagar and Waranagal.
Evidence of ancient gold workings has been The Indian League was started in 1875 by
found in the Kolar fields in the pre-Christian Sisir Kumar Ghosh with the object of
centuries. The Satavahanas may have used “stimulating the sense of nationalism
gold as bullion, for they did not issue gold amongst the people” and of encouraging
coins as the Kushans did. They issued political education.
mostly coins of lead, which is found in the Source: Beginning of Modern Nationalism in
Deccan. India, page No. 271-272
Statement 2 is correct
The people of Deccan knew the art of paddy 32. Consider the following pairs:
transplantation, and in the first two
centuries the area between the Krishna and Form of people’s Resistance
the Godavari, especially at the mouths of the resistance
river, formed a great rice bowl. The people of
the Deccan also produced cotton. 1. Tribal Uprising Sanyasi
Statement 3 is correct Revolt
The official language of the Satavahanas
was Prakrit. All inscriptions were composed 2. Civil Uprising Chuar
in this language and written in the Brahmi Uprising
script, as was the case in Ashokan times.
One Prakrit text called Gathasattasai or the 3. Peasant Uprising Moplah
Gathasaptasati is attributed to a Uprising
Satavahana king called Hala.
Which of the pairs given above is/are
Source:
correctly matched?
Ancient India (Ram Sharan Sharma, class
(a) 1 and 2 only
XI), Chapter 16, Page 156, 160
(b) 3 only
(c) 2 and 3 only
31. Which of the following association was
formed by Badruddin Tyabji and (d) 1, 2 and 3
Pherozshah Mehta? Explanation:
(a) The Poona Sarvajanik Sabha
Form of people’s Resistance
(b) The Bombay Presidency resistance
Association
(c) The Madras Mahajan Sabha 1. Tribal Uprising Chuar Uprising
(d) The Indian League
Explanation:
Option (b) is correct: 2. Civil Uprising Sanyasi Revolt
Political Associations in Bombay
The Poona Sarvajanik Sabha was founded 3. Peasant Moplah Uprising
in 1867 by Mahadeo Govind Ranade and Uprising
others, with the object of serving as a bridge
between the government and the people. Source: A Brief History of Modern India,
14

Page 152 and 170


Page

DELHI: VIJAY NAGAR 9717380832 & OLDRAJENDER NAGAR 9811293743 | JAIPUR: 8290800441
BENGALURU: KORMANGALA 7619166663 & CHANDRA LAYOUT 7619136663 | BHOPAL: 7509975361
PATNA: 7463950774 | INDORE: 7314977441 | RANCHI: 7463950774 | www.ksgindia.com
TEST - 1122 (EXPLANATION & SOURCE)
33. Consider the following: of monks and nuns according to rules
1. Chakiarkoothu attributed to the Buddha.
2. Koodiyattam Source:
3. Krishnattam https://www.britannica.com/topic/Vinaya-
4. Ramanattam Pitaka
Which of the above are performing arts
of Kerala? 35. With reference to Wardha Basic
(a) 2 and 3 only Education Scheme, consider the
following statements:
(b) 1 and 2 only
1. It was formulated by
(c) 3 and 4 only
Dr.S.Radhakrishnan.
(d) 1, 2, 3 and 4
2. ‘Learning through activity’ was the
Explanation: basic principle behind it.
Option (d) is correct: Which of the Statements given above
Kerala is the home of several traditional is/are correct?
dance and dance - drama forms, the most (a) 1 only
notable being Kathakali. Kathakali, as a
(b) 2 only
dance form popular today, is considered to
be of comparatively recent origin. However, (c) Both 1 and 2
it is an art which has evolved from many (d) Neither 1 nor 2
social and religious theatrical forms which Explanation:
existed in the southern region in ancient Statement 1 is incorrect: The Congress
times. Chakiarkoothu, Koodiyattam, had organised a National Conference on
Krishnattam and Ramanattam are few of the Education in October 1937 in Wardha. In
ritual performing arts of Kerala which have the light of the resolutions passed there,
had a direct influence on Kathakali in its Zakir Hussain committee formulated a
form and technique. Legend has it that the detailed national scheme for basic education
refusal of the Zamorin of Calicut to send his based on Gandhi's ideas.
Krishnattam troupe to Travancore, so Statement 2 is correct: The main principle
enraged the Raja of Kottarakkara, that he behind this scheme was ‘learning through
was inspired to compose the Ramanattam. activity’. It was based on Gandhi’s ideas
Source:http://ccrtindia.gov.in/kathakali.p published in a series of articles in the weekly
hp Harijan. Gandhi thought that Western
education had created a gulf between the
34. The Vinaya Pitaka is best described by educated few and the masses and had also
which of the following? made the educated elite ineffective.
(a) One of the revered texts of Jainism Source: A Brief History of India; Chapter:
which contains rules for daily Chapter 30: Development of Education;
conduct of lay Jain followers. Page: 640
(b) A Jain monument in Bihar.
(c) A Buddhist text that regulates 36. Which among the following were the
monastic life and the daily major reasons behind the civil
affairs of monks and nuns. uprisings during the British rule?
(d) None of the above 1. New socio-economic changes
Explanation: Option (c) is correct during the colonial rule that went
against the people.
Vinaya Piṭaka, (Pāli and Sanskrit: “Basket of
Discipline”), the oldest and smallest of the 2. The ruin of Indian handicraft
three sections of the Buddhist canonical industries.
Tipiaka (“Triple Basket”) and the one that 3. The priestly classes instigated
15

regulates monastic life and the daily affairs hatred and rebellion against alien
rule.
Page

DELHI: VIJAY NAGAR 9717380832 & OLDRAJENDER NAGAR 9811293743 | JAIPUR: 8290800441
BENGALURU: KORMANGALA 7619166663 & CHANDRA LAYOUT 7619136663 | BHOPAL: 7509975361
PATNA: 7463950774 | INDORE: 7314977441 | RANCHI: 7463950774 | www.ksgindia.com
TEST - 1122 (EXPLANATION & SOURCE)
Select the correct answer using the milk and grain were given as offerings.
code given below. Elaborate rituals were followed during the
(a) 1 and 2 only worship.
(b) 2 and 3 only Source: class11 History/TN Board/ Pg. 31
(c) 1 and 3 only
(d) 1, 2 and 3 38. The Mural paintings at Badami were
Explanation: All the statements are patronized by which of the following
correct dynasties:
Under the Company rule, there were rapid (a) Pandava Dynasty
changes in the economy, administration and (b) Chola Dynasty
land revenue system that went against the (c) Pallava Dynasty
people. (d) Chalukya dynasty
The ruin of Indian handicraft industries due Explanation: Option (d) is correct
to colonial policies impoverished millions of Badami was the capital of the early
artisans whose misery was further Chalukyan dynasty which ruled the region
compounded by the disappearance of their from 543 to 598 CE. With the decline of the
traditional patrons and buyers—princes, Vakataka rule, the Chalukyas established
chieftains, and zamindars. their power in the Deccan. TheChalukya
The priestly classes instigated hatred and king, Mangalesha, patronised the excavation
rebellion against alien rule, because the of the Badami caves. He was the younger son
religious preachers, priests,pundits, of the Chalukya king, PulakesiI, and the
maulvis, etc., had been dependent on the brother of Kirtivarman I. The Inscription in
traditional landed and bureaucratic elite. Cave No.4 mentions the date 578–579 CE,
The fall of zamindars and feudal lords describes the beauty of the cave and
directly affected the priestly class. includes the dedication of the image
Source: A Brief History of Modern India, ofVishnu. Thus it may be presumed that the
Page 151 and 152 cave was excavated in the same era and the
patron records his Vaishnava
37. With reference to Rig Vedic Aryans, affiliation.Therefore, the cave is popularly
consider the following statements: known as the Vishnu Cave. Only a fragment
of the painting has survived on the vaulted
1. They worshiped the natural
roof of the front mandapa.Paintings in this
forces.
cave depict palace scenes. One shows
2. Worship of idols in temples was Kirtivarman, the son ofPulakesi I and the
prominent. elder brother of Bangladesh, seated inside
Which of the statements given above the palace with his wife and feudatories
are correct? watching a dance scene. Towards the corner
(a) 1 only of the panel are figures of Indra and his
(b) 2 only retinue.
(c) Both 1 and 2 Source: An Introduction to Indian Art,
(d) Neither 1 nor 2 NCERT XI, Chapter 5, Page- 62
Explanation:
Statement 1 is correct: The Rig Vedic 39. Consider the following statements with
Aryans worshiped the natural forces like reference to the reign of Harsha:
earth, fire, wind, rain and thunder. They 1. As a devout Buddhist, Harsha
personified these natural forces into many convened a grand assembly at
gods and worshipped them. Kannauj to widely publicize the
Statement 2 is incorrect: There were no doctrines of Mahayana.
temples and no idol worship during the early 2. The practice of praising the patron
16

Vedic period. Prayers were offered to the initiated by Harishena in the time
gods in the expectation of rewards. Ghee, of Samudragupta became
Page

DELHI: VIJAY NAGAR 9717380832 & OLDRAJENDER NAGAR 9811293743 | JAIPUR: 8290800441
BENGALURU: KORMANGALA 7619166663 & CHANDRA LAYOUT 7619136663 | BHOPAL: 7509975361
PATNA: 7463950774 | INDORE: 7314977441 | RANCHI: 7463950774 | www.ksgindia.com
TEST - 1122 (EXPLANATION & SOURCE)
common and well established 3. Image worship and rituals
under Harsha. Which of the above is/ are the
Which of the statements given above features/ features of Mahayana
is/are correct? Buddhism?
(a) 1 only (a) 1 only
(b) 2 only (b) 1 and 2 only
(c) Both 1 and 2 (c) 2 and 3 only
(d) Neither 1 nor 2 (d) 1, 2 and 3
Explanation: Explanation: Option (d) is correct
Statement 1 and Statement 2 are correct- Deification of the Buddha was a major
Harsha followed a tolerant religious policy. A feature of Mahayana Buddhism. Buddha
Shaiva in his early years, he gradually was seen as God.
became a great patron of Buddhism. As a Mahayana talks a great deal about the
devout Buddhist he convened a grand bodhisattva (the 'enlightenment being') as
assembly at Kanauj to widely publicize the being the ideal way for a Buddhist to live.
doctrines of Mahayana. The assembly was Anyone can embark on the bodhisattva path.
attended not only by Hsuan Tsang and the This is a way of life, a way of selflessness; it
Kamarupa ruler Bhaskaravarman, but also is a deep wish for all beings, no matter who
by the kings of twenty states and by several they are, to be liberated from suffering.
thousand priests belonging to different
Image worship and rituals are also features
sects. Two thatched halls were built to
of Mahayana Buddhism. Theravada
accommodate 1000 persons each. However,
Buddhism worships Buddha in the form of
the most important construction was a huge
symbols only.
tower in the middle of which a golden statue
of the Buddha, as tall as the king himself, Source: Ancient India by R S Sharma
was placed. Harsha worshipped the image https://www.bbc.co.uk/religion/religions/
and gave a public dinner. The discussion in buddhism/subdivisions/mahayana.shtml
the conference was initiated by Hsuan Tsang
who dilated on the virtues of Mahayana 41. Which of the following was/were the
Buddhism and challenged the audience to provisions of Indian Councils Act
refute his arguments. 1892?
Banabhatta gives us a flattering account of 1. Imperial Legislative Council could
the early years of his patron in his book not have non-officials members.
Harshacharita in an ornate style which 2. Budget could be discussed.
became a model for later writers. Harsha is
3. Supplementary questions can be
remembered not only for his patronage and
asked.
learning but also for the authorship of three
plays: Priyadarshika, Ratnavali, and Select the correct answer using the
Nagananda. Bana attributes great poetical code given below:
skill to him, and some later authors consider (a) 1 and 3 only
him to have been a literary monarch. The (b) 2 only
practice of praising the patron initiated by (c) 2 and 3 only
Harishena in the time of Samudragupta (d) 1, 2 and 3
became common and well established under
Explanation:
Harsha.
Statement 1 is incorrect: Number of
Source-India’s Ancient Past, R.S. Sharma,
additional members in Imperial Legislative
Chapter-27,Page 304
Councils and the Provincial Legislative
Councils was raised. In the Imperial
40. Consider the following statements: Legislative Council, now the governor-
17

1. Deification of the Buddha general could have ten to sixteen non-


2. Treading the path of Bodhisattvas officials (instead of six to ten previously).
Page

DELHI: VIJAY NAGAR 9717380832 & OLDRAJENDER NAGAR 9811293743 | JAIPUR: 8290800441
BENGALURU: KORMANGALA 7619166663 & CHANDRA LAYOUT 7619136663 | BHOPAL: 7509975361
PATNA: 7463950774 | INDORE: 7314977441 | RANCHI: 7463950774 | www.ksgindia.com
TEST - 1122 (EXPLANATION & SOURCE)
Statement 2 is correct: Budget could be Banaras.The stone was transported from
discussed. The budget could not be voted Mathura and Chunar to the various sites
upon, nor could any amendments be made where the pillars have been found and here
to it. the stone was cut and carried by craftsmen.
Statement 3 is incorrect: Questions could The uniformity of style in the pillar capitals
be asked but Supplementary could not be suggests that they were all sculpted by
asked, nor could answers to any question be craftsmen belonging from the same region.
discussed. Statement 3 is correct:
Source: Modern History/ M.Spectrum/ pg. Stupa, vihara and chaitya are part of
279 Buddhist and Jain Monastic complexes but
the largest number belongs to the Buddhist
42. In context of art of Mauryan period religion. One of the best examples of the
consider the following statements: structure of a stupa in the third century BCE
is at Bairat in Rajasthan. It is a very grand
1. The Mauryan pillars were
stupa having a circular mound with a
constructed in pieces by a mason.
circumambulatory path. The great stupa at
2. The stone pillars were sculpted by Sanchi (which will be discussed later) was
craftsmen belonging from the built with bricks during the time of Ashoka
same region. and later it was covered with stone and many
3. Stupa, Vihara and Chaitya are new additions were made.
part of Buddhist and Jain Source: Ch-3, Arts of Maurya Period,An
Monastic complexes Introduction toIndian Art Part-I, Class-
Which of the statements given above XI.http://theindianhistory.org/Mauryan/m
is/are correct? auryan-pillars.html
(a) 1 and 2 only
(b) 2 and 3 only 43. Consider the following statements:
(c) 1 and 3 only 1. Effort was made to keep the army
(d) 1, 2 and 3 away from the civilian population.
Explanation: 2. The number of Indian soldiers was
Statement 1 is incorrect: drastically reduced even as the
The tradition of constructing pillars is very number of European soldiers was
old and it may be observed that erection of increased.
pillars was prevalent in the Achaemenid 3. Communal and regional loyalties
Empire as well. But the Mauryan pillars are were encouraged among the
different from the Achaemenid pillars. The soldiers.
Mauryan pillars are rock-cut pillars thus Which of the above changes were
displaying the carver’s skills, whereas the brought as a post-1857 revolt
Achaemenid pillars are constructed in pieces measure?
by a mason. Stone pillars were erected all (a) 1 and 2 only
over the Mauryan Empire with inscriptions (b) 2 and 3 only
engraved on them.
(c) 1 and 3 only
Statement 2 is correct:
(d) 1, 2 and 3
Pillars erected by Ashoka are one of the
Explanation:
finest remains of the Mauryan art. The
pillars with Ashoka proclamations inscribed Option (d) is correct:
on them were placed or located either in The Army, which was at the forefront of the
sacred enclosures or in the vicinity of towns. outbreak, was thoroughly reorganized and
These pillars consisted of two types of stone- British military policy came to be dominated
the spotted red and white sandstone from by the idea of “division and counterpoise”.
The British could no longer depend on
18

the region of Mathura and the beige colored


fine-grained hard sandstone usually with Indian loyalty, so the number of Indian
small black spots quarried in Chunar near soldiers was drastically reduced even as the
Page

DELHI: VIJAY NAGAR 9717380832 & OLDRAJENDER NAGAR 9811293743 | JAIPUR: 8290800441
BENGALURU: KORMANGALA 7619166663 & CHANDRA LAYOUT 7619136663 | BHOPAL: 7509975361
PATNA: 7463950774 | INDORE: 7314977441 | RANCHI: 7463950774 | www.ksgindia.com
TEST - 1122 (EXPLANATION & SOURCE)
number of European soldiers was increased. could have had a larger army only if he was
The concept of divide and rule was adopted in a position to mobilize the support of all his
with separate units being created on the feudatories in the time of war.
basis of caste/community/region. Recruits Source-India’s Ancient Past, R.S.Sharma,
were to be drawn from the martial races of Chapter-27,Page 301
Punjab, Nepal, and north-western frontier
who had proved loyal to the Britishduring
45. Saddler Commission was related to:
the Revolt. Effort was made to keep the army
away from the civilian population. (a) Civil services reform
Source: A Brief History of Modern India, (b) University Education
Page 187 and 188 (c) Introduction of Female Education
(d) Decentralization of municipal
44. With reference to the rule of Harsha, administration.
consider the following statements: Explanation: Option (b) is correct
1. The authority of Harsha was The Saddler University Commission (1917-
limited only to North India 19) was set up to study and report on
excluding Kashmir. problems of Calcutta University but its
2. Harshavardhana’s empire saw the recommendations were applicable more or
decline of feudalism. less to other universities also. It reviewed the
entire field from school education to
Which of the statements given above
university education. It held the view that,
is/are correct?
for the improvement of university education,
(a) 1 only improvement of secondary education was a
(b) 2 only necessary pre-condition.
(c) Both 1 and 2 Source: A Brief History of India; Chapter:
(d) Neither 1 nor 2 Chapter 30: Development of Education;
Explanation: Page: 638
Statement 1 is correct and Statement 2 is
incorrect-Harsha is called the last great 46. Which of the following pair(s) is/are
Hindu emperor of India, but he was neither correctly matched?
a staunch Hindu nor the ruler of the whole Sultanat Department
country. His authority was limited to north e Term of
India excluding Kashmir. Rajasthan,
Punjab, UP, Bihar, and Orissa were under 1. Diwan-i- Military
his direct control, but his sphere of influence Arz
spread over a much wider area. It appears 2. Diwan-i- Religious
that the peripheral states acknowledged his Risalat Matters
sovereignty. In eastern India he faced 3. Diwan-i- State
opposition from the Shaivite king Insha Corresponden
Shashanka of Gauda, who felled the Bodhi ce
tree at Bodh-Gaya
Select the correct answer using the
Harshavardhana’s reign is an example of
code given below:
transition from ancient to medieval times.
Harsha governed his empire on the same (a) 1 and 3 only
lines as did the Guptas, but his (b) 2 and 3 only
administration had become feudal and (c) 3 only
decentralized. It is stated that Harsha had (d) 1, 2 and 3
100,000 horses and 60,000 elephants. This Explanation: Option (d) is correct
appears astonishing because the Mauryas,
The most important department of state,
who ruled over virtually the entire country
next to the wazir’s was the diwan i-arz or the
19

except the Deep South, maintained only


military department. The head of this
30,000 cavalry and 9000 elephants. Harsha
department was called the ariz-i-mamalik.
Page

DELHI: VIJAY NAGAR 9717380832 & OLDRAJENDER NAGAR 9811293743 | JAIPUR: 8290800441
BENGALURU: KORMANGALA 7619166663 & CHANDRA LAYOUT 7619136663 | BHOPAL: 7509975361
PATNA: 7463950774 | INDORE: 7314977441 | RANCHI: 7463950774 | www.ksgindia.com
TEST - 1122 (EXPLANATION & SOURCE)
The ariz was not the commander-in-chief of among Indian troops and the Indian
the army, since the sultan himself prisoners of war (POWs) and to incite anti-
commanded all the armed forces. British feelings among the people of these
There were two other important departments countries. One mission under Raja
of state: the diwan-i-risalat and the diwan-i- Mahendra Pratap Singh, Barkatullah and
insha. The former dealt with religious Obaidullah Sindhi went to Kabul to organise
matters, pious foundations and stipends to a ‘provisional Indian government’ there with
deserving scholars and men of piety. It was the help of the crown prince, Amanullah.
presided over by the chief sadr, who was Source: Modern History/ M.Spectrum/ pg.
generally a leading qazi.The diwan-i-insha 324-325
dealt with state correspondence. All the
correspondence, formal or confidential, 48. Consider the following states:
between the ruler and the sovereigns of other
1. Gujarat
states, and with his subordinate officials was
dealt with by this department. 2. Maharashtra
Source-History of Medieval India,Satish 3. Chhattisgarh
Chandra,Chapter 7 ,Page- 137-138 4. Madhya Pradesh
Pithoro mural paintings are associated
47. Consider the following statements: with which of the above state(s)?
1. The Berlin Committee for Indian (a) 1 only
Independence was established (b) 3 only
with the help of the German (c) 1 and 2 only
foreign office under ‘Zimmerman (d) 1, 2, 3 and 4
Plan’. Explanation: Option (a) is correct
2. Raja Mahendra Pratap Singh went We observe mural painting on interior and
to Kabul to organise a ‘provisional exterior walls of houses in villages or havelis
Indian government’ there with the in different parts of the country.These
help of the crown prince, paintings are usually made by women either
Amanullah. at the time of ceremonies or festivals or as a
Which of the statements given above routine to clean and decorate the walls.
is/are correct? Some of the traditional forms of murals are
(a) 1 only pithoro in parts of Rajasthan and
(b) 2 only Gujarat,Mithila painting in northern Bihar’s
(c) Both 1 and 2 Mithila region, warli paintings in
Maharashtra, or simply paintings on the
(d) Neither 1 nor 2
walls, be it in a village of Odisha or Bengal,
Explanation: MadhyaPradesh or Chhattisgarh.
Statement 1 is correct: The Berlin Source:An Introduction to Indian Art,
Committee for Indian Independence was NCERT XI, Chapter 5, Page- 67
established in 1915 by Virendranath
Chattopadhyay, Bhupendranath Dutta, Lala
Hardayal and others with the help of the 49. Consider the following statements:
German foreign office under ‘Zimmerman 1. Brahmo Samaj discarded the
Plan’. These revolutionaries aimed to doctrine of karma and
mobilise the Indian settlers abroad to send transmigration of soul.
volunteers and arms to India to incite 2. Tattvabodhini Sabha was devoted
rebellion among Indian troops there and to to the systematic study of India’s
even organise an armed invasion of British past with a rational outlook.
India to liberate the country. 3. Dharma Sabha stood for the
Statement 2 is correct: The Indian preservation of the status quo in
20

revolutionaries in Europe sent missions to socio-religious matters however


Baghdad, Persia, Turkey and Kabul to work
Page

DELHI: VIJAY NAGAR 9717380832 & OLDRAJENDER NAGAR 9811293743 | JAIPUR: 8290800441
BENGALURU: KORMANGALA 7619166663 & CHANDRA LAYOUT 7619136663 | BHOPAL: 7509975361
PATNA: 7463950774 | INDORE: 7314977441 | RANCHI: 7463950774 | www.ksgindia.com
TEST - 1122 (EXPLANATION & SOURCE)
favoured the promotion of Western Explanation:
education, even for girls. Option (d) is correct:
Which of the statements given above The Bangabhasha Prakasika Sabha was
is/are correct? formed in 1836 by associates of Raja
(a) 1 and 2 only Rammohan Roy.
(b) 2 and 3 only The Zamindari Association, more
(c) 1 and 3 only popularly known as the ‘Landholders’
(d) 1, 2 and 3 Society’, was founded to safeguard the
interests of the landlords. Although limited
Explanation:
in its objectives, the Landholders’ Society
Statement 1 is incorrect: Brahmo Samaj marked the beginning of an organized
took no definite stand on the doctrine of political activity and use of methods of
karma and transmigration of soul and left it constitutional agitation for the redressal of
to individual Brahmos to believe either grievances.
way.
The Bengal British India Society was
Statement 2 is correct: Debendranath founded in 1843 with the object of “the
Tagore headed the Tattvabodhini Sabha collection and dissemination of information
(founded in 1839) which, along with its relating to the actual condition of the people
organ TattvabodhiniPatrika in Bengali, was of British India... and to employ such other
devoted to the systematic study of India’s means of peaceful and lawful character as
past with a rational outlook and to the may appear calculated to secure the welfare,
propagation of Rammohan’s ideas. extend the just rights and advance the
Statement 3 is correct: It was an orthodox interests of all classes of our fellow subjects”.
society, which stood for the preservation of In 1851, both the Landholders’ Society and
the status quo in socio-religious matters, the Bengal British India Society merged into
opposing even the abolition of the British Indian Association. It sent a
sati.However, it favoured the promotion of petition to the British Parliament demanding
Western education, even for girls. inclusion of some of its suggestions in the
Source: A Brief History of Modern India, renewed Charter of the Company
Page 230, 232, 247 Source: Beginning of Modern Nationalism in
India, page No. 270
50. Which of the following statements are
correct regarding political associations 51. Which of the following personalities
before Indian national congress? was linked with the Ghadr party?
1. The Bangabhasha Prakasika 1. LalaHardayal
Sabha was formed in 1836 by 2. Ramchandra
associates of Raja Rammohan
3. Bhagwan Singh
Roy.
4. BhikajiCama
2. The Zamindari Association was
popularly known as the 5. Barkatullah
‘Landholders’ Society’. 6. Bhai Parmanand
3. The Landholders’ Society and the Select the correct answer using the
Bengal British India Society code given below:
merged into the British Indian (a) 1, 2 and 4 only
Association. (b) 2, 3 and 5 only
Select the correct answer using the (c) 1, 2, 4 and 6 only
code given below: (d) 1, 2, 3, 5 and 6
(a) 1 and 2 only Explanation:
(b) 2 and 3 only Option (d) is correct: The Ghadr Party was
21

(c) 1 and 3 only a revolutionary group organised around a


(d) 1, 2 and 3 weekly newspaper The Ghadr with its
Page

DELHI: VIJAY NAGAR 9717380832 & OLDRAJENDER NAGAR 9811293743 | JAIPUR: 8290800441
BENGALURU: KORMANGALA 7619166663 & CHANDRA LAYOUT 7619136663 | BHOPAL: 7509975361
PATNA: 7463950774 | INDORE: 7314977441 | RANCHI: 7463950774 | www.ksgindia.com
TEST - 1122 (EXPLANATION & SOURCE)
headquarters at San Francisco and Samhita, Brahmana, Aranyaka, and
branches along the US coast and in the Far Upanishad.
East officials, publish revolutionary and Source: India’s Ancient Past, R.S. Sharma,
anti-imperialist literature, work among Chp31,Page-333
Indian troops stationed abroad, procure
arms and bring about a simultaneous revolt
in all British colonies. The moving spirits
behind the Ghadr Party were LalaHardayal,
Ramchandra, Bhagwan Singh, Kartar 53. With reference to Town Planning in the
Singh Saraba, Barkatullah, and Bhai Harappan Culture, consider the
Parmanand. The Ghadrites intended to following statements:
bring about a revolt in India. Their plans 1. The large stones had been used in
were encouraged by two events in 1914—the almost all kinds of constructions.
KomagataMaru incident and the outbreak of 2. Harappa, Mohenjodaro and
the First World War. Kalibangan each had its own
BhikajiCama was not part of this citadel built on a high podium of
programme. mud brick.
Source: Modern History/ M.Spectrum/ pg. 3. The floor of the Great Bath of
323 Mohenjodaro was made of burnt
bricks.
52. Which of the following can be cited as Which of the statements given above
‘smriti literature’: is/are correct?
1. Puranas (a) 1 and 3 only
2. Vedanga (b) 2 only
3. Vedas (c) 2 and 3 only
4. Dharmashastra (d) 1, 2 and 3
5. Nitishastra Explanation:
Select the correct answer using the Statement 1 is incorrect: The large-scale
code given below: use of burnt bricks in almost all kinds of
(a) 1, 2, 4 and 5 only constructions and the absence of stone
buildings are the important characteristics
(b) 1, 2, 3 and 4 only
of the Harappan culture. Another
(c) 3 only remarkable feature was the underground
(d) 1, 2, 3, 4 and 5 drainage system connecting all houses to the
Explanation: Option (a) is correct street drains which were covered by stone
In the Hindu tradition, the Vedas have the slabs or bricks.
status of shruti (literally, ‘that which has Statement 2 is correct: The Harappan
been heard’). They are thought to embody an culture was distinguished by its system of
eternal, self-existent truth realized by the town planning on the lines of the grid system
rishis(seers) in a state of meditation or – that is streets and lanes cutting across one
revealed to them by the gods. The category another almost at right angles thus dividing
of smriti(literally, ‘remembered’) texts the city into several rectangular blocks.
includes the Vedanga, Puranas, epics, Harappa, Mohenjodaro and Kalibangan
Dharmashastra, and Nitishastra. The word each had its own citadel built on a high
Veda comes from the root vid (literally, ‘to podium of mud brick. Below the citadel in
know’) and means ‘knowledge’.There are four each city lay a lower town containing brick
Vedas—Rig, Sama, Yajur, and Atharva. The houses, which were inhabited by the
Rig Veda contains the world's oldest common people.
surviving poetry, some of it of extraordinary Statement 3 is correct: The most important
beauty and philosophical depth.Each Veda
22

public place of Mohenjodaro is the Great


has four parts, the last three of which Bath measuring 39 feet length, 23 feet
sometimes blend into each other—the breadth and 8 feet depth. Flights of steps at
Page

DELHI: VIJAY NAGAR 9717380832 & OLDRAJENDER NAGAR 9811293743 | JAIPUR: 8290800441
BENGALURU: KORMANGALA 7619166663 & CHANDRA LAYOUT 7619136663 | BHOPAL: 7509975361
PATNA: 7463950774 | INDORE: 7314977441 | RANCHI: 7463950774 | www.ksgindia.com
TEST - 1122 (EXPLANATION & SOURCE)
either end lead to the surface. There are side amongst the people” and of encouraging
rooms for changing clothes. The floor of the political education.
Bath was made of burnt bricks. Water was Source: Beginning of Modern Nationalism in
drawn from a large well in an adjacent room, India, page No. 271-272
and an outlet from one corner of the Bath led
to a drain. It must have served as a ritual
bathing site. The largest building in
Mohenjodaro is a granary measuring 150 55. With reference to the period between
feet length and 50 feet breadth. But in the 800 CE and 1000 CE, consider the
citadel of Harappa we find as many as six following statements:
granaries 1. The Palas and Pratiharas
Source: class11 History/TN Board/Pg. 19 struggled to gain control over the
area extending from Banaras to
south Bihar.
54. Which among the following
2. Dharmapala revived the Nalanda
associations were formed by
University by granting villages for
Surendranath Banerjea and Ananda
meeting its expenses.
Mohan Bose?
Which of the statements given above
(a) The Poona Sarvajanik Sabha
is/are correct?
(b) The Madras Mahajan Sabha
(a) 1 only
(c) Indian National Association
(b) 2 only
(d) The Indian League
(c) Both 1 and 2
Explanation: Option c is correct:
(d) Neither 1 nor 2
Political Associations in Bombay
Explanation:
The Poona Sarvajanik Sabha was founded
Statement 1 is correct- The period
in 1867 by Mahadeo Govind Ranade and
following the death of Harsha was a period of
others, with the object of serving as a bridge
political confusion.For some time,
between the government and the people.
Lalitaditya, the ruler of Kashmir brought the
The Bombay Presidency Association was Punjab under his control and even
started by Badruddin Tyabji, Pherozshah controlled Kanauj which, since the days of
Mehta and K.T. Telang in 1885. Harsha, was considered the symbol of the
Political Associations in Madras sovereignty of north India—a position
The Madras Mahajan Sabha was founded in whichDelhi was to acquire later. Control of
1884 by M. Viraraghavachari, B. Kanauj also implied control of theupper
Subramaniya Aiyer and P. Anandacharlu. Gangetic valley and its rich resources in
The Indian Association of Calcutta (also trade and agriculture.Lalitaditya even
known as the Indian National Association) invaded Bengal or Gaud, and killed its
superseded the Indian League and was reigning king. But his power waned with the
founded in 1876 by younger nationalists of rise of the Palas and the Gurjara-
Bengal led by Surendranath Banerjea and Pratiharas.ThePalas and the Pratiharas
Ananda Mohan Bose, who were getting clashed with each other for the control of
discontented with the conservative and pro- the area extending from Banaras to south
landlord policies of the British Indian Bihar which again had rich resources and
Association. The Indian Association was the well developed imperial traditions. The
most important of pre-Congress associations Pratiharas also clashed with
and aimed to “promote by every legitimate theRashtrakutas of the Deccan.
means the political, intellectual and material Statement 2 is correct- Thus, for about a
advancement of the people.” hundred years, from the middle of the eighth
The Indian League was started in 1875 by to the middle of the ninth century, the Pala
Sisir Kumar Ghosh with the object of rulers dominated eastern India. For Some
23

“stimulating the sense of nationalism time, their control extended upto Varanasi.
Their power is attested to by an Arab
Page

DELHI: VIJAY NAGAR 9717380832 & OLDRAJENDER NAGAR 9811293743 | JAIPUR: 8290800441
BENGALURU: KORMANGALA 7619166663 & CHANDRA LAYOUT 7619136663 | BHOPAL: 7509975361
PATNA: 7463950774 | INDORE: 7314977441 | RANCHI: 7463950774 | www.ksgindia.com
TEST - 1122 (EXPLANATION & SOURCE)
merchant, Sulaiman, who visited India in Founded in 1887 at Lahore by Shiv Narayan
the middle of the ninth century, and wrote Agnihotri(1850-1927), earlier a Brahmo
an account of it. He calls the Pala Kingdom follower, DevSadan is a religious and social
Ruhma, (or Dharma, short for Dharmapala), reform society. The society emphasised on
and says that the Pala ruler was at war with the eternity of the soul, the supremacy of the
his neighbours, the Pratiharas and the guru, and the need for good action.
Rashtrakutas, but his troops were more A Parsi social reformer, Behramji M.
numerous that his adversaries. He tells us Malabari (18531912), founded the
that it was customary for the Pala king to be SevaSadan in 1908 along with a friend,
accompanied by a force of 50,000 elephants, DiwanDayaramGidumal. Malabari spoke
and that 10,000– 15,000 men in his army vigorously against child marriage and for
were employed ‘in fulling and washing widow remarriage among Hindus.
clothes’. Even if these figures may be The Arya Samaj Movement, revivalist in
exaggerated, we can assume that the Palas form though not in content, was the result of
had a large military force at their disposal. a reaction to Western influences. Its founder,
But we do not know whether they had a large DayanandaSaraswati or Mulshankar (1824-
standing army, or whether their forces 1883) was born in the old Morvi state in
consisted largely of feudal levies. Gujarat in a brahmin family. He wandered
Information about the Palas is also provided as an ascetic for fifteen years (1845-60) in
to us by Tibetan chronicles, although these search of truth. The first Arya Samaj unit
were written in the seventeenth century. was formally set up by him at Bombay in
According to these, the Pala rulers were 1875 and later the headquarters of the
great patrons of Buddhist learning and Samaj were established at Lahore.
religion. The Nalandauniversity which
Narayan Malhar Joshi founded the Social
had been famous all over the eastern
Service League in Bombay (1911) with an
world was revived by Dharmapala, and
aim to secure for the masses better and
200 villages were set apart for meeting its
reasonable conditions of life and work. They
expenses. He also founded the
organised many schools, libraries, reading
Vikramasilauniversity which became second
rooms, day nurseries and cooperative
only to Nalanda in fame. It was located on
societies.
the top of a hill, on the banks of the Ganga
in Magadha, amidst pleasant surroundings. Source:A Brief History of Modern India, Page
The Palas built many viharas in which a 239, 244, 246, 247
large number of Buddhist monks lived.
Source:History of Medieval India, Satish 57. With reference to Religion of Later
Chandra, Chapter-2, Page- 28 Vedic Period, consider the following
statements:
56. Arrange the formation of following 1. Indra was the most popular god of
socio-religious reform movements in this period.
chronological order: 2. The importance of sacrifices
1. DevSadan declined and that of prayers
increased.
2. Arya Samaj
Which of the statements given above
3. SevaSadan
is/are correct?
4. Social Service League
(a) 1 only
Select the correct answer using the
(b) 2 only
code given below:
(c) Both 1 and 2
(a) 1-2-3-4
(d) Neither 1 nor 2
(b) 2-1-3-4
Explanation:
(c) 2-1-4-3
Statement 1 is incorrect:Gods of the
24

(d) 1-2-4-3
Early Vedic period like Indra and Agni lost
Explanation: their importance.Prajapathi (the creator),
Page

DELHI: VIJAY NAGAR 9717380832 & OLDRAJENDER NAGAR 9811293743 | JAIPUR: 8290800441
BENGALURU: KORMANGALA 7619166663 & CHANDRA LAYOUT 7619136663 | BHOPAL: 7509975361
PATNA: 7463950774 | INDORE: 7314977441 | RANCHI: 7463950774 | www.ksgindia.com
TEST - 1122 (EXPLANATION & SOURCE)
Vishnu (the protector) and Rudra (the 1898— Repressive laws under IPC Section
destroyer) became prominent during the 124 A were further amplified with new
Later Vedic period. provisions under IPC Section 156 A 1899—
Statement 2 is incorrect: Sacrifices were Number of Indian members in Calcutta
still important and the rituals connected Corporation were reduced.
with them became more elaborate. The 1904— Official Secrets Act curbed
importance of prayers declined and that freedom of press.
of sacrifices increased. Priesthood became 1904— Indian Universities Act ensured
a profession and a hereditary one. The greater government control over universities,
formulae for sacrifices were invented and which
elaborated by the priestly class. Therefore, it described as factories producing political
towards the end of this period there was a revolutionaries.
strong reaction against priestly domination
Growth of Education
and against sacrifices and rituals. The rise of
Buddhism and Jainism was the direct result While, on the one hand, the spread of
of these elaborate sacrifices. Also, the education led to an increased awareness
authors of the Upanishads, which is the among the masses, on the other hand, the
essence of Hindu philosophy, turned away rise in unemployment and
from the useless rituals and insisted on true underemployment among the educated drew
knowledge (jnana) for peace and salvation. attention to poverty and the underdeveloped
state of the country’s economy under
Source: class11 History/TN Board/Pg. 33
colonial rule. This added to the already
simmering discontent among the more
58. Which among the following are the radical nationalists.
reasons for rise in revolutionary International Influences
activities?
Remarkable progress made by Japan after
1. The spread of education led to an 1868 and its emergence as an industrial
increased awareness among the power opened the eyes of Indians to the fact
masses. that economic progress was possible even in
2. Official Secrets Act curbed an Asian country without any external help.
freedom of press. The defeat of the Italian army by Ethiopians
3. Remarkable progress made by (1896), the Boer wars (1899-1902) where the
Japan after independence. British faced reverses and Japan’s victory
Select the correct answer using the over Russia (1905) demolished myths of
codes given below: European invincibility.
(a) 1 and 2 only Source: Era of Militant Nationalism (1905-
(b) 2 and 3 only 1909), page No. 289
(c) 1 and 3 only
(d) 1, 2 and 3 59. Which among the following were the
anti-partition campaigns under
Explanation:
moderates?
Option (d) is correct:
1. They gave the slogan Political
The nationalists were wide awake to the fact freedom is the lifebreath of a
that instead of giving more rights to the nation.
Indians, the government was taking away
2. People fasted, bathed in the Ganga
even the existing ones.
and walked barefoot in
1892— The Indian Councils Act was processions singing Bande
criticised by nationalists as it failed to satisfy Mataram.
them.
3. People tied rakhis on each other’s
1897— The Natu brothers were deported hands as a symbol of unity of the
25

without trial and Tilak and others, two halves of Bengal.


imprisoned on charges of sedition.
Page

DELHI: VIJAY NAGAR 9717380832 & OLDRAJENDER NAGAR 9811293743 | JAIPUR: 8290800441
BENGALURU: KORMANGALA 7619166663 & CHANDRA LAYOUT 7619136663 | BHOPAL: 7509975361
PATNA: 7463950774 | INDORE: 7314977441 | RANCHI: 7463950774 | www.ksgindia.com
TEST - 1122 (EXPLANATION & SOURCE)
Select the correct answer using the Explanation: Option (d) is correct
code given below: Bhagat Singh, Sukhdev and Rajguru were
(a) 1 and 2 only tried and given capital punishment in the
(b) 2 and 3 only Lahore conspiracy case.
(c) 1 and 3 only InMeerut Conspiracy case ,MuzafrAhamed,
(d) 1, 2 and 3 S.A Dange, etc. were arrested on the charge
of conspiracy to overthrow the British
Government of India through strikes and
other militant methods.
Explanation: Muzaffarpur Conspiracy was by the
Statement 1 is incorrect Khudiran Bose and PrafulChaki to kill the
The militant nationalists tried to transform Chief Presidency Magistrate DH Kingsford of
the antipartition and Swadeshi Movement Muzaffarpur.
into a mass struggle and gave the slogan of Delhi Conspiracy case was organized to
India’s independence from foreign rule. assassinate the then Viceroy of India, Lord
“Political freedom is the lifebreath of a Hardinge. It was headed by Rashbehari
nation,” declared Aurobindo. Thus, the Bose.
Extremists gave the idea of India’s Source: Modern India, Bipan Chandra
independence the central place in India’s
politics. The goal of independence was to be
61. Which of the following pairs is/are
achieved through self-sacrifice.
correctly matched?
Statements 2 and 3 are correct
Mughal Soldier Meaning
October 16, 1905, the day the partition
1. Ahadis Individual Trooper
formally came into force, was observed as a
day of mourning throughout Bengal. People 2. Baraq-andaz Bowmen
fasted, bathed in the Ganga and walked 3. Walashahis Musketeers
barefoot in processions singing Bande Select the correct answer using the
Mataram (which almost spontaneously code given below:
became the theme song of the movement). (a) 1 only
‘Amar Sonar Bangla’, the national anthem of (b) 2 and 3 only
present-day Bangladesh, was composed by
(c) 3 only
Rabindranath Tagore, and was sung by huge
crowds marching in the streets. (d) 1, 2 and 3
People tied rakhis on each other’s hands as Explanation:Option (a) is correct
a symbol of unity of the two halves of Bengal. The cavalry, was the principal arm of the
Later in the day, Surendranath Banerjea and Mughal army and the mansabdars provided
Ananda Mohan Bose addressed huge the overwhelming proportion of it. In
gatherings (perhaps the largest till then addition to the mansabdars, the Mughal
under the nationalist banner). Within a few emperors used to entertain individual
hours of the meeting, Rs 50,000 was raised troopers, called ahadis. The ahadis have
for the movement. been called gentlemen-troopers and received
Source: Era of Militant Nationalism (1905- much higher salaries than other troopers.
1909), page No. 293 They were a highly trusted corps, being
recruited directly by the emperors and
having their own muster-master.
60. Bhagat Singh, Sukhdev and Rajguru
An ahadi mustered up to five horses, though
were given capital punishment in
sometimes two of them shared a horse. The
which of the following cases?
duties of ahadis were of a miscellaneous
(a) Meerut Conspiracy case character. Most clerks of the imperial offices,
(b) Delhi Conspiracy Case the painters of the court, the foramen in the
26

(c) Muzaffarpur Conspiracy Case royal karlshamnas belonged to this corps.


(d) Lahore Conspiracy case Many were appointed as adjutants and
Page

DELHI: VIJAY NAGAR 9717380832 & OLDRAJENDER NAGAR 9811293743 | JAIPUR: 8290800441
BENGALURU: KORMANGALA 7619166663 & CHANDRA LAYOUT 7619136663 | BHOPAL: 7509975361
PATNA: 7463950774 | INDORE: 7314977441 | RANCHI: 7463950774 | www.ksgindia.com
TEST - 1122 (EXPLANATION & SOURCE)
carriers of imperial orders. In Shah Jahan’s (b) B.R Ambedkar
reign, they numbered 7000 and were often (c) Jyotirao Phule
sent into the fighting line, where they were (d) None of the above
well distributed over the different parts of the
Explanation: Option (b) is correct
army. Many of them worked as skilled
musketeers (baraq-andaz) and bowmen Through the liberation movements such as
(tir-andaz).In addition to the ahadis, the Mahad Satyagraha (1927) and Kalaram
emperors maintained a corps of royal Temple-entry-struggle (1930), Dr. Ambedkar
bodyguards (walashahis) and armed palace attempted to change the Hindu mindset. The
guards. They were cavalrymen but served on aim of Mahad Satyagraha was to unfurl “the
foot in the citadel and the palace. banner of equality” and to smash the “steel
Source-History of Medieval India,Satish frames of the caste system”.
Chandra, Chapter 15 ,Page- 310 Source:
http://www.mea.gov.in/Images/attach/am
62. With reference to the India House an b/Volume_17_01.pdf
Indian Home Rule Society, consider
the following statements: 64. Consider the following statement with
1. It was started by Shyamji Krishna reference to the Khaljis:
varma in London in 1905. 1. Jalaluddin Khalji followed a policy
2. Madanlal Dhingra assassinated of tolerance and believed that
the India office bureaucrat state should be based on the
Curzon-Wyllie in 1909. willing support of the governed.
Which of the statements given above 2. Alauddin, continued with his
is/are correct? uncle’s policy and avoided harsh
(a) 1 only punishments to nobles and non-
Muslims.
(b) 2 only
Which of the statements given above
(c) Both 1 and 2
is/are correct?
(d) Neither 1 nor 2
(a) 1 only
Explanation:
(b) 2 only
Statement 1 is correct: Shyamji Krishna
(c) Both 1 and 2
varma had started in London in 1905 an
Indian Home Rule Society—‘India House’— (d) Neither 1 nor 2
as a centre for Indian students, a Explanation:
scholarship scheme to bring radical youth Statement 1 is correct and Statement 2 is
from India, and a journal The Indian incorrect- Jalaluddin Khalji ruled only for a
Sociologist. Revolutionaries such as brief period of six years. He tried to mitigate
Savarkar and Hardayal became the some of the harsh aspects of Balban’s rule.
members of India House. He was the first ruler of the Delhi Sultanate
Statement 2 is correct:MadanlalDhingra to clearly put forward the view that the state
from this circle assassinated the India office should be based on the willing support of the
bureaucrat Curzon-Wyllie in 1909. Soon, governed, and that since the large majority
London became too dangerous for the of the people in India were Hindus, the state
revolutionaries, particularly after Savarkar in India could not be a truly Islamic state. He
had been extradited in 1910 and transported also tried to gain the goodwill of the nobility
for life in the Nasik conspiracy case. by a policy of tolerance and avoiding harsh
Source: Modern History/ Spectrum/ pg. punishments. However, many people,
321 including his supporters, considered this to
be a weak policy which was not suited to the
times. The Delhi Sultanate faced numerous
63. Who of the following led “Mahad
27

internal and external foes, and for this


Satyagrah”? reason there was a sense of insecurity.
(a) Mahatma Gandhi
Page

Jalaluddin’s policy was reversed by Alauddin


DELHI: VIJAY NAGAR 9717380832 & OLDRAJENDER NAGAR 9811293743 | JAIPUR: 8290800441
BENGALURU: KORMANGALA 7619166663 & CHANDRA LAYOUT 7619136663 | BHOPAL: 7509975361
PATNA: 7463950774 | INDORE: 7314977441 | RANCHI: 7463950774 | www.ksgindia.com
TEST - 1122 (EXPLANATION & SOURCE)
who awarded drastic punishments to all historical narrative of their victories.
those who dared to oppose him. That is why we know a great deal more about
Source-History of Medieval India, Satish the Cholas that their predecessors.
Chandra, Chapter 6,Page- 98 Source-History of Medieval India, Satish
Chandra, Chapter 3,Page 42

65. Consider the following statements with 66. With reference to ‘Tholu Bommalata’,
reference to the rulers of Chola consider the following statements:
Empire: 1. They are the traditional Rod
1. Rajaraja and Rajendra I marked puppet of Karnataka.
their victories by erecting a 2. The theme of the puppet plays are
number of Siva and Jain temples drawn from the Ramayana,
at various places. Mahabharata and Puranas.
2. The Chola rulers adopted the Which of the statements given above
practice of installing pillar is/are correct?
inscriptions in the temples which (a) 1 only
gave a historical narrative of their (b) 2 only
victories.
(c) Both 1 and 2
Which of the statements given above
(d) Neither 1 nor 2
is/are correct?
Explanation:
(a) 1 only
Statement 1 is incorrect and Statement 2
(b) 2 only
is correct-Tholu Bommalata, Andhra
(c) Both 1 and 2 Pradesh's shadow theatre has the richest
(d) Neither 1 nor 2 and strongest tradition. The puppets are
Explanation: large in size and have jointed waist,
Statement 1 and Statement 2 are shoulders, elbows and knees. They are
incorrect-The greatest Chola rulers were coloured on both sides. Hence, these
Rajaraja (985–1014) and his son Rajendra I puppets throw coloured shadows on the
(1014–1044). Rajaraja destroyed the Chera screen. The music is dominantly influenced
navy at Trivandrum, and attacked Quilon. by the classical music of the region and the
He then conquered Madurai and captured theme of the puppet plays are drawn from
the Pandyan king. He also invaded Sri Lanka the Ramayana, Mahabharata and Puranas.
and annexed its northern part to his empire. Source:http://ccrtindia.gov.in/puppetform
These moves were partly motivated by his s.php
desire to bring the trade with the Southeast
Asian countries under his control. The 67. Consider the following statements with
Coromandel coast and Malabar were the reference to Chandragupta II:
centres for India’s trade with the countries of
1. The court of Chandragupta II at
Southeast Asia. One of his naval exploits
Ujjain was adorned by numerous
was the conquest of the Maldives. Rajaraja,
scholars including Kalidasa and
annexed the northwestern parts of the
Amarasimha.
Ganga kingdom in Karnataka, and overran
Vengi. 2. It was during Chandragupta’s
reign that the Chinese pilgrim Fa-
Rajaraja and Rajendra I marked their
hsien visited India.
victories by erecting a number of Siva and
Vishnu temples at various places. The Which of the statements given above
most famous of these was the Brihadishwara is/are correct?
temple at Tanjore which was completed in (a) 1 only
1010. The Chola rulers adopted the (b) 2 only
28

practice of having inscriptions written on (c) Both 1 and 2


the walls of these temples, giving a (d) Neither 1 nor 2
Page

DELHI: VIJAY NAGAR 9717380832 & OLDRAJENDER NAGAR 9811293743 | JAIPUR: 8290800441
BENGALURU: KORMANGALA 7619166663 & CHANDRA LAYOUT 7619136663 | BHOPAL: 7509975361
PATNA: 7463950774 | INDORE: 7314977441 | RANCHI: 7463950774 | www.ksgindia.com
TEST - 1122 (EXPLANATION & SOURCE)
Explanation: included in Sangam literature. The
Statement 1 and statement 2 are correct- Tolkappiyam is essentially awork on
Chandragupta II adopted the title of grammar, but it also includes a discussion
Vikramaditya, which had been first used by of phonology, semantics, syntax, and literary
an Ujjain ruler in 58–57 BC as a mark of conventions.
victory over the SakaKshatrapas of western A number of Tamil didactic works were
India. This Ujjain ruler is traditionally called written in the post-5th century period. The
Shakari or the enemy of the Shakas. The most famous of these is
VikramSamvat or era was started in 58– 57 Tiruvalluvar’sTirukkural, a work on
BC by Shakari. However, Chandragupta II ethics, polity, and love (5th–6th
proved to be a greater Shakari and centuries). Of the several Tamil epics, two
Vikramaditya. The court of Chandragupta of the best known are the
II at Ujjain was adorned by numerous SilappadikaramandManimekalai. The
scholars including Kalidasa and former is a little earlier that the latter, but
Amarasimha.It was during both were composed in about the 5th–6th
Chandragupta’s reign that the Chinese centuries CE.
pilgrim Fa-hsien (AD 399–414) visited Source: India’s Ancient Past, R.S. Sharma,
India and wrote an elaborate account of the Chp3,Page-37
life of its people.
Source-India’s Ancient Past, R.S. Sharma, 69. Consider the following:
Chapter-24, Page 276
1. Nuniz
2. Pelsaert
68. Which of the following pairs is/are
3. De Laet
correctly matched?
4. Nicoloi Conti
Sangam Theme
Which of the above travellers visited
Literature
the Indian Subcontinent during the
1. Tolkappiyam Grammar Mughal period?
2. Tirukkural Ethics (a) 1 and 2 only
3. Manimekalai Epic (b) 2 and 3 only
Select the correct answer using the (c) 3 and 4 only
code given below:
(d) 1, 2, 3 and 4
(a) 1 and 3 only
Explanation: Option (b) is correct
(b) 2 and 3 only
During the Mughal period, many European
(c) 3 only traders and travellers came to India, and
(d) 1, 2 and 3 some of them have left accounts about the
Explanation: Option (d) is correct social and economic conditions of the
The Sangam corpus includes six of the eight country. In general, they have emphasized
anthologies of poems included in the the wealth and prosperity of India and the
Ettutokai (The Eight Collections), and nine ostentatious life-style of the ruling classes,
of the ten pattus (songs) of the on the one hand, and on the other the
Pattuppattu(The Ten Songs). The style and grinding poverty of the ordinary people—the
certain historical references in the poems peasants, the artisans and the labourers.
suggest that they were composed between Babur was struck by the scanty clothes worn
the 3rd century BCE and the 3rd century by the ordinary people. He observed that
CE. They were compiled into anthologies in ‘peasants and people of low standing go
about the mid-8th century. A few centuries about naked.’ He then goes on to describe
later, these anthologies were collected into the langota or decency cloth worn by men,
the super-anthologies (i.e., anthologies of and the sari worn by women. His impression
anthologies)called the Ettuthokai and the has been corroborated by later European
29

Pattuppattu. The earliest parts of the first travellers. Ralph Fitch, who came to India
two books of theTolkappiyam can also be towards the end of the sixteenth century,
Page

DELHI: VIJAY NAGAR 9717380832 & OLDRAJENDER NAGAR 9811293743 | JAIPUR: 8290800441
BENGALURU: KORMANGALA 7619166663 & CHANDRA LAYOUT 7619136663 | BHOPAL: 7509975361
PATNA: 7463950774 | INDORE: 7314977441 | RANCHI: 7463950774 | www.ksgindia.com
TEST - 1122 (EXPLANATION & SOURCE)
says that at Banaras ‘the people go naked support. Tilak’sKesari and Maharatta had
save a little cloth bound about their middle.’ been campaigning for the strike for months.
De Laet wrote that the labourers had Source: A Brief History of Modern India;
insufficient clothing to keep themselves Chapter 32: The Movement of the
warm and cozy during winter. Working Class; Page: 658
Regarding food, rice, millet and pulses (what 71. “He taught at the Hindu College from
Pelsaert and De Laet called khicheri) 1826 to 1831, was the leader and
formed the staple diet, along with fish in inspirer of a progressive trend.
Bengal and the coasts, and meat in the Drawing inspiration from the great
southern peninsula. In north India, chapatis French Revolution, he inspired his
made of wheat or coarse grains, with pulses pupils to think freely and rationally,
and green vegetables were common. The question all authority, love liberty,
ordinary people, it is said, ate their main equality and freedom, and oppose
meal in the evening, and chewed pulse or decadent customs and traditions. His
other parched grain in the day. Ghee and oil followers also supported women’s
were much cheaper than food grains then, rights and education.”
and seem to have been a staple part of the Which one of the following
poor man’s food.But salt and sugar were personalities has been described in the
more expensive. above passage?
Source-History of Medieval India, Satish (a) Henry Vivian Derozio
Chandra, Chapter 16 ,Page- 314
(b) Ishwar Chandra Vidyasagar
(c) Keshab Chandra Sen
70. Consider the following pairs:
(d) JyotibaPhule
Newspaper Founder/Editor Explanation:
Option (a) is correct:
Bharat Sasipada During the late 1820s and early 1830s, there
Shramjeevi Banerjea emerged a radical, intellectual trend among
the youth in Bengal, which came to be
Deenbandhu Narain Meghajee known as the ‘Young Bengal Movement’. A
Lokhanday young Anglo-Indian, Henry Vivian Derozio
(1809-31), who taught at the Hindu College
Maharatta BalGangadharTil from 1826 to 1831, was the leader and
ak inspirer of this progressive trend. Drawing
inspiration from the great French
Which of the pairs given above is/are Revolution, Derozio inspired his pupils to
correctly matched? think freely and rationally, question all
(a) 1 only authority, love liberty, equality and freedom,
and oppose decadent customs and
(b) 1 and 3 only
traditions. The Derozians also supported
(c) 2 and 3 only women’s rights and education. Also, Derozio
(d) 1, 2 and 3 was perhaps the first nationalist poet of
Explanation: modern India.
Pair 1 is correctly matched: In 1870, Source: A Brief History of Modern India,
SasipadaBanerjea started a workingmen’s Page 235
club and newspaper Bharat Shramjeevi.
Pair 2 is correctly matched: In 1880, 72. One of the three principles of Jainism,
NarainMeghajeeLokhanday started the also known as Triratnas is ‘Right
newspaperDeenbandhu and set up the Conduct’. Right conduct refers to the
Bombay Mill and Millhands Association. observance of the five great vows,
30

Pair 3 is correctly matched: In 1899, the which of the following are part of it?
first strike by the Great Indian Peninsular 1. Not to injure life
Page

Railways took place, and it got widespread


DELHI: VIJAY NAGAR 9717380832 & OLDRAJENDER NAGAR 9811293743 | JAIPUR: 8290800441
BENGALURU: KORMANGALA 7619166663 & CHANDRA LAYOUT 7619136663 | BHOPAL: 7509975361
PATNA: 7463950774 | INDORE: 7314977441 | RANCHI: 7463950774 | www.ksgindia.com
TEST - 1122 (EXPLANATION & SOURCE)
2. Not to lie 1. Indians were not allowed to
3. Not to steal participate in the election of
4. Not to acquire property legislative councils.
5. Not to lead immoral life 2. Separate electorates for Muslims
for election to the central council
Which of the statements given above
were established.
is/are correct?
Which of the statements given above
(a) 1, 2 and 3 only
is/are correct?
(b) 4 and 5 only
(a) 1 only
(c) 1, 2, 3 and 4 only
(b) 2 only
(d) 1, 2, 3, 4 and 5
(c) Both 1 and 2
Explanation:
(d) Neither 1 nor 2
Option (d) is correct: The three principles
Explanation:
of Jainism, also known as Triratnas (three
Statement 1 is incorrect: The elective
gems), are:
principle was recognised for the non-official
· right faith membership of the councils in India. Indians
· right knowledge were allowed to participate in the election of
· right conduct. various legislative councils, though on the
Right faith is the belief in the teachings and basis of class and community.
wisdom of Mahavira. Right Knowledge is the Statement 2 is correct: For the first time,
acceptance of the theory that there is no God separate electorates for Muslims for election
and that the world has been existing without to the central council were established—a
a creator and that all objects possess a soul. most detrimental step for India
Right conduct refers to the observance of the Source: Modern History/ M.Spectrum/ pg.
five great vows: 309
· not to injure life
· not to lie 74. With reference to the Servants of India
· not to steal Society,consider the following
· not to acquire property statements:
· not to lead immoral life. 1. It was founded by Dadabhai
Both the clergy and laymen had to strictly Naoroji.
follow the doctrine of ahimsa. Mahavira 2. It aimed to prepare a cadre of
regarded all objects, both animate and selfless workers who were to
inanimate, have souls and various degrees of devote their lives to the cause of
consciousness. They possess life and feel the country in a religious spirit.
pain when they are injured. Mahavira 3. The society chose to remain aloof
rejected the authority of the Vedas and from political activities and
objected to the Vedic rituals. He advocated a organizations like the Indian
very holy and ethical code of life. Even the National Congress.
practice of agriculture was considered sinful Which of the statements given above
as it causes injury to the earth, worms and is/are correct?
animals. Similarly the doctrine of asceticism (a) 1 and 3 only
and renunciation was also carried to
(b) 2 and 3 only
extreme lengths by the practice of starvation,
nudity and other forms of self-torture. (c) 2 only
Source: class11 History/TN Board/Pg. 39 (d) 1, 2 and 3
Explanation:
73. With reference to Morley-Minto Statement 1 is incorrect: Gopal Krishna
Gokhale (1866-1915), a liberal leader of the
31

Reforms, consider the following


statements: Indian National Congress, founded the
Page

DELHI: VIJAY NAGAR 9717380832 & OLDRAJENDER NAGAR 9811293743 | JAIPUR: 8290800441
BENGALURU: KORMANGALA 7619166663 & CHANDRA LAYOUT 7619136663 | BHOPAL: 7509975361
PATNA: 7463950774 | INDORE: 7314977441 | RANCHI: 7463950774 | www.ksgindia.com
TEST - 1122 (EXPLANATION & SOURCE)
Servants of India Society in 1905 with the fields. The mahasabha could distribute
help of M.G. Ranade. new lands, and exercise ownership rights
Statement 2 and 3 are correct:The aim of over them. It could also raise loans for the
the society was to train national village and levy taxes.
missionaries for the service of India; to Source-History of Medieval
promote, by all constitutional means, the India,SatishChandra,Chapter 3,Page 47
true interests of the Indian people; and to
prepare a cadre of selfless workers who 76. In the context of Indian history, the
were to devote their lives to the cause of principle of “Dyarchy (diarchy)” refers
the country in a religious spirit. to:
The society chose to remain aloof from (a) Division of the central legislature
political activities and organisations like into two houses.
the Indian National Congress.
(b) Introduction of double
Source: A Brief History of Modern India, government i.e., Central and State
Page 239 governments.
(c) Having two sets of rulers; one in
75. Consider the following statements with London and another in Delhi.
reference to the Chola Government: (d) Division of the subjects
1. Agraharams were villages with delegated to the provinces into
Brahman settlements in which two categories.
most of the land was rent-free. Explanation:
2. The Mahasabha had the power to Option (d) is correct:
tax and distribute new lands with
Dyarchy, also spelled diarchy, system of
their ownership.
double government introduced by
Which of the statements given above the Government of India Act (1919) for the
is/are correct? provinces of British India. It marked the first
(a) 1 only introduction of the democratic principle into
(b) 2 only the executive branch of the British
(c) Both 1 and 2 administration of India.
(d) Neither 1 nor 2 The principle of dyarchy was a division of the
Explanation: executive branch of each provincial
government into authoritarian and
Statement 1 and Statement 2 are correct-
popularly responsible sections. The first was
The ur was a general assembly of the village.
composed of executive councillors,
However, we know more about the working
appointed, as before, by the crown. The
of the mahasabha. This was a gathering of
second was composed of ministers who were
the adult men in the Brahman villages which
chosen by the governor from the elected
were called agraharams. These were
members of the provincial legislature. These
villages with Brahman settlements in
latter ministers were Indians.
which most of the land was rent-free.
These villages enjoyed a large measure of Source:
autonomy. The affairs of the village were https://www.britannica.com/topic/dyarchy
managed by an executive committee to
which educated persons owning property 77. Arrange the following events in
were elected either by drawing lots or by chronological order:
rotation. These members had to retire every 1. Imprisoned of Natu brother
three years. There were other committees for
2. The Indian Councils Act 1892
helping in the assessment and collection of
land revenue, for maintenance of law and 3. Official Secrets Act
order, justice, etc. One of the important 4. Indian Universities Act
32

committees was the tank committee which Select the correct answer using the
looked after the distribution of water to the code given below:
Page

DELHI: VIJAY NAGAR 9717380832 & OLDRAJENDER NAGAR 9811293743 | JAIPUR: 8290800441
BENGALURU: KORMANGALA 7619166663 & CHANDRA LAYOUT 7619136663 | BHOPAL: 7509975361
PATNA: 7463950774 | INDORE: 7314977441 | RANCHI: 7463950774 | www.ksgindia.com
TEST - 1122 (EXPLANATION & SOURCE)
(a) 1, 2, 3, 4 cotton. Surplus grain is stored in granaries.
(b) 2, 1, 3, 4 Animals like sheep, goats and buffalo were
(c) 2, 1, 4, 3 domesticated. The use of horse is not yet
firmly established. A number of other
(d) 1, 2, 4, 3
animals were hunted for food including deer.
Explanation:
Statement 2 is correct: Specialized groups
Option (b) is correct: of artisans include goldsmiths, brick
➢ 1892— The Indian Councils Act was makers, stone cutters, weavers, boat-
criticised by nationalists as it failed to builders and terracotta manufacturers.
satisfy them. Bronze and copper vessels are the
➢ 1897— The Natu brothers were outstanding examples of the Harappan
deported without trial and Tilak and metal craft. Gold and silver ornaments are
others, imprisoned on charges of found in many places. Pottery remains plain
sedition. and in some places red and black painted
➢ 1898— Repressive laws under IPC pottery is found. Beads were manufactured
Section 124 A were further amplified from a wide variety of semi-precious stones.
with new provisions under IPC Section Statement 3 is correct: Internal trade was
156 A extensive with other parts of India. Foreign
➢ 1899— Number of Indian members in trade was mainly conducted with
Calcutta Corporation were reduced. Mesopotamia, Afghanistan and Iran Gold,
➢ 1904— Official Secrets Act curbed copper, tin and several semi-precious stones
freedom of press. were imported. Main exports were several
agricultural products such as wheat, barley,
➢ 1904— Indian Universities Act ensured
peas, oil seeds and a variety of finished
greater government control over
products including cotton goods, pottery,
universities, which it described as
beads, terracotta figures and ivory products.
factories producing political
There is much evidence to prove the trade
revolutionaries.
links between the Indus and Sumerian
Source: Modern History/ M.Spectrum/ pg. people. Many seals of Indus valley have
288 been found in Mesopotamia. Trade was of
the barter type. The seals and the terracotta
78. With reference to Economic life in the models of the Indus valley reveal the use of
Harappan Culture, consider the bullock carts and oxen for land transport
following statements: and boats and ships for river and sea
1. Wheat and barley were the main transport.
crops grown. Source: class11 History/TN Board/Pg. 20-
2. Red and black painted pottery is 21
found in some places.
3. Many seals of the Indus valley 79. Which of the following best describes
have been found in Mesopotamia. the ‘tribhanga posture?
Which of the statements given above (a) Synchronization of hand moments
are correct? with legs and facial expressions
(a) 1 and 2 only (b) A stance where the body is bent
(b) 1 and 3 only in three directions.
(c) 2 and 3 only (c) A dance scene in which three
(d) 1, 2 and 3 artists synchronise their
movements.
Explanation:
(d) None of the above
Statement 1 is correct: There was a great
progress in all spheres of economic activity Explanation:
33

such as agriculture, industry and crafts and Option (b) is correct:


trade. Wheat and barley were the main The term tribhanga, can be broken down
Page

crops grown besides sesame, mustard and from the Sanskrit root: tri or three and
DELHI: VIJAY NAGAR 9717380832 & OLDRAJENDER NAGAR 9811293743 | JAIPUR: 8290800441
BENGALURU: KORMANGALA 7619166663 & CHANDRA LAYOUT 7619136663 | BHOPAL: 7509975361
PATNA: 7463950774 | INDORE: 7314977441 | RANCHI: 7463950774 | www.ksgindia.com
TEST - 1122 (EXPLANATION & SOURCE)
bhanga or bends – literally translating to ● differentiated between the labour in
‘three bends’. It is an architectural term used the Indian Owned Factories and those
to describe Indian sculptures, art, and in the British-ownedfactories;
iconography. It is also a term in dance theory ● believed that labour legislations would
used to describe a stance where the body is affect the competitive edge enjoyed by
bent in three directions. In either sculptures the Indian-ownedindustries;
or in dance, the figure stands in a posture in ● did not want a division in the
which the head is inclined to one direction, movement on the basis of classes;
the upper body is bent in the opposite
● did not support the Factory Acts of
direction, and the lower body, below the
1881 and 1891for these reasons.
waist, takes on a reverse direction. These
movements are dramatic energetic muscular Thus, earlier attempts to improve the
actions! economic conditions of the workers were in
the nature of the philanthropic efforts which
The thrice-bent figure is an image found
were isolated, sporadic and aimed at specific
throughout Hindu temples where the
local grievances.
sculptures around the temple are in a
standing posture where the body follows an Source: A Brief History of Modern India;
‘S’ curve. The tribhanga pose in an instance Chapter 32: The Movement of the
captures the vitality, elegance, and Working Class; Page: 658
sensuousness of Indian art. The sculptures
embody the ideal of beauty transposed from 81. Arrange the following Mughal
a worldly and lay plane to attain a spiritual Emperors in the correct chronological
dimension! order:
Source:Ch-7 Indian Bronze Sculpture,An 1. Farrukhsiyar
Introduction toIndian Art Part-I, Class- 2. Jahandar Shah
XI.http://tribhanga.tripod.com/TSD.html
3. Bahadur Shah I
Select the correct answer using the
80. With reference to the efforts of the code given below:
early nationalists for the working-class
(a) 1-2-3
movements, which of the following
statements is/are correct? (b) 2-3-1
1. The moderates were sensitive with (c) 3-2-1
the issues of the working classes. (d) 2-1-3
2. No differentiation between the Explanation: Option (c) is correct
labourers of the Indian and Aurangzeb was the last illustrious king of
British-owned factories. Mughal empire. The Mughal emperors who
3. Early nationalists support the succeeded him were known as later
Factory Acts of 1881 and 1891. Mughals. Their list is given below.
Select the correct answer using the Bahadur Shah I (1707 AD-1712 AD)
codes given below: Aurangzeb died in 1707 AD and he was
(a) 1 and 2 only succeeded by his son prince Muazzam who
(b) 2 and 3 only sat on the throne with the title, Bahadur
Shah I. He tried to reverse the orthodox
(c) 3 only
policies of his father, Aurangzeb. He also
(d) None of the Above tried to have friendly relations with Sikhs
Explanation: Statements 1, 2 and 3 are and Marathas. He died in 1712.
not correct: Jahandar Shah (1712-1713)
The early nationalists, especially the Bahadur Shah was succeeded by his son
Moderates, Jahandar Shah. He married a dancing girl
● were indifferent to the labour’s cause; who became his queen. He was attacked and
34

killed by his nephew and son-in-law


Farrukhsiyar.
Page

DELHI: VIJAY NAGAR 9717380832 & OLDRAJENDER NAGAR 9811293743 | JAIPUR: 8290800441
BENGALURU: KORMANGALA 7619166663 & CHANDRA LAYOUT 7619136663 | BHOPAL: 7509975361
PATNA: 7463950774 | INDORE: 7314977441 | RANCHI: 7463950774 | www.ksgindia.com
TEST - 1122 (EXPLANATION & SOURCE)
Farrukhsiyar (1713-1719) very little ornamentation whereas the
He became the king of Mughal kingdom with Mathura Buddha images continue to depict
the help of Sayyid brothers, the two Mughal folds of the drapery in the Buddha images
Vazirs named Syed Abdullah Khan Barha and the halo around the head is profusely
and Syed Hassan Ali Khan decorated. One can visit museums at
Barha.Farrukhsiyar behaved like a mere Mathura, Sarnath, Varanasi, New Delhi,
puppet in the hands of Sayyid brothers and Chennai, Amaravati, etc. to study the
when he resisted their control on him, he features of early sculptures.
was deposed and executed by the latter. Mathura artists mainly used red sandstone
Source-History of Medieval India, Satish to sculpt images.
Chandra, Chapter 18 ,Page- 363 Source:Ch-4,Post-Mauryan Trends in
Indian Art and Architecture, An Introduction
82. In context of Mathura School, consider toIndian Art Part-I, Class-XI.
the following statements:
1. The Buddha image at Mathura is 83. At Mohenjo-daro, the stone statue of a
modelled on the lines of earlier bust bearded priest was made up of
Yaksha images which of the following material?
2. The sculptural tradition in (a) Limestone
Mathura had the confluence of (b) Granite
Bactria and Parthia. (c) Red sandstone
3. Buddha images at Mathura are (d) Steatite
made up of bronze using lost wax Explanation:
technique.
Option (d) is correct:
Which of the statements given above
The stone statues found at Harappa and
is/are correct?
Mohenjodaro are excellent examples of
(a) 1 only handling three dimensional volumes. In
(b) 2 and 3 only stone are two male figures— one is a torso in
(c) 1 and 2 only red sandstone and the other is a bust of a
(d) 1 and 3 only bearded man in steatite—which are
Explanation: extensively discussed.
Statement 1 is correct and Statement 2 is Source: Ch-2, Arts of the Indus Valley, An
incorrect: Introduction toIndian Art Part-I, Class-XI.
The sculptural tradition in Gandhara had
the confluence of Bactria, Parthia and the 84. With reference to Partition of Bengal,
local Gandhara Tradition. The local consider the following statements:
sculptural tradition at Mathura became so 1. Bengali was reduced to minorities
strong that the tradition spread to other in Bengal itself.
parts of northern India. The best example in 2. Partition was annulled later.
this regard is the stupa sculptures found at Which of the statements given above
Sanghol in the Punjab. The Buddha image at is/are correct?
Mathura is modelled on the lines of earlier
(a) 1 only
Yaksha Images whereas in Gandhara it has
Hellenistic features (b) 2 only
Statement 3 is incorrect: (c) Both 1 and 2
The traditional centre, Mathura, remained (d) Neither 1 nor 2
the main art production site whereas Explanation:
Sarnath and Kosambi also emerged as The British government’s decision to
important centres of art production. Many partition Bengal had been made public in
35

Buddha images in Sarnath have plain December 1903. The idea was to have two
transparent drapery covering both provinces: Bengal comprising Western
shoulders, and the halo around the head has Bengal as well as the provinces of Bihar and
Page

DELHI: VIJAY NAGAR 9717380832 & OLDRAJENDER NAGAR 9811293743 | JAIPUR: 8290800441
BENGALURU: KORMANGALA 7619166663 & CHANDRA LAYOUT 7619136663 | BHOPAL: 7509975361
PATNA: 7463950774 | INDORE: 7314977441 | RANCHI: 7463950774 | www.ksgindia.com
TEST - 1122 (EXPLANATION & SOURCE)
Orissa, and Eastern Bengal and Assam. Explanation:
Bengal retained Calcutta as its capital, while Both the statements are correct-Babur’s
Dacca became the capital of Eastern Bengal. advent into India was significant from many
The official reason given for the decision was points of view. For the first time since the
that Bengal with a population of 78 million downfall of the Kushan empire, Kabul and
(about a quarter of the population of British Qandharbecame integral parts of an empire
India) had become too big to be comprising north India. Since these areas
administered. It was also stated that has always acted as staging places for an
partition would help in the development of invasion of India, by dominating them Babur
Assam if it came under the direct jurisdiction and his successors were able to give to India
of the government. This was true to some security from external invasions for almost
extent, but the real motive behind the 200 years. Economically also, the control
partition plan was seen to be the British ofKabul and Qandhar strengthened India's
desire to weaken Bengal, the nerve centre of foreign trade since these two towns were the
Indian nationalism. starting points for caravans meant for China
Statement 1 is correct: This it sought to in the east, and the
achieve by putting the Bengalis under two Mediterranean seaports in the west. Thus,
administrations by dividing them: India could take a greater share in the great
(a) on the basis of language, thus reducing trans-Asian trade.In north India, Babur
the Bengalis to a minority in Bengal smashed the power of the Lodis and the
itself (as in the new proposal Bengal Rajput Confederacy led by Rana Sanga.
proper was to have 17 million Bengalis Thereby, he destroyed the balance of power
and 37 million Hindi and Oriya in the area. This was a long step towards the
speakers); and establishment of an all-India empire.
(b) on the basis of religion, as the western However, a number of conditions had still to
half was to be a Hindu majority area (42 be fulfilled before this could be achieved.
million out of a total 54 million) and the Babur introduced a new mode of warfare in
eastern half was to be a Muslim majority India. Although gunpowder was known in
area (18 million out of a total of 31 India earlier, Babur showed what a skilled
million) combination of artillery and cavalry could
Statement 2 is correct: Because of mass achieve. His victories led to rapid
political protests, popularisation of gunpowder and artillery in
the partition was annulled in 1911. India. Since artillery was expensive, it
Source: Modern History/ M.Spectrum/ pg. favoured those rulers who had large
291-292 resources at their command. Hence the era
of small kingdoms ended.
Source-History of Medieval India, Satish
85. With reference to the advent of Babur,
Chandra, Chapter 12 ,Page 225-226
consider the following statements:
1. The share of the Indian
Subcontinent in the great trans- 86. With reference to Punjab’s extremism,
Asian trade was increased under consider the following statements:
Babur. 1. LalaLajpat Rai organised the
2. He introduced a new mode of extremist Anjuman-i-Mohisban-i-
warfare based on artillery and Watan in Lahore.
cavalry. 2. Bhai Parmanand brought out
Which of the statements given above Bharat Mata with its motto of self-
is/are correct? help at any cost.
(a) 1 only Which of the statements given above
is/are correct?
(b) 2 only
(a) 1 only
36

(c) Both 1 and 2


(b) 2 only
(d) Neither 1 nor 2
Page

(c) Both 1 and 2


DELHI: VIJAY NAGAR 9717380832 & OLDRAJENDER NAGAR 9811293743 | JAIPUR: 8290800441
BENGALURU: KORMANGALA 7619166663 & CHANDRA LAYOUT 7619136663 | BHOPAL: 7509975361
PATNA: 7463950774 | INDORE: 7314977441 | RANCHI: 7463950774 | www.ksgindia.com
TEST - 1122 (EXPLANATION & SOURCE)
(d) Neither 1 nor 2 1. It was introduced in the 15th
Explanation: century A.D by
Both the statements are incorrect: The MahapurushaSankaradeva.
Punjab extremism was fuelled by issues 2. It was introduced as a medium for
such as frequent famines coupled with rise propagation of the Vaishnava
in land revenue and irrigation tax, practice faith.
of ‘begar’ by zamindars and by the events in Select the correct answer using the
Bengal. Among those active here were code given below:
LalaLajpat Rai who brought out Punjabee (a) Manipuri
(with its motto of self-help at any cost) and (b) Bharatnatyam
Ajit Singh (Bhagat Singh’s uncle) who
(c) Sattriya
organised the extremist Anjuman-i-
Mohisban-i-Watan in Lahore with its (d) None of the Above
journal, Bharat Mata. Before Ajit Singh’s Explanation: Option (C) is correct
group turned to extremism, it was active in The Sattriya dance form was introduced in
urging non-payment of revenue and water the 15th century A.D by the great Vaishnava
rates among Chenab colonists and Bari saint and reformer of Assam,
Doab peasants. Other leaders included Aga MahapurushaSankaradeva as a powerful
Haidar, Syed Haider Raza, Bhai Parmanand medium for propagation of the Vaishnava
and the radical Urdu poet, Lalchand ‘Falak’. faith. The dance form evolved and expanded
Source: Modern History/ M.Spectrum/ pg. as a distinctive style of dance later on. This
320-321 neo-Vaishnava treasure of Assamese dance
and drama has been, for centuries, nurtured
and preserved with great commitment by the
87. Which of the following statements
Sattras i.e. Vaishnava maths or
is/are correct?
monasteries. Because of its religious
1. Lord Dalhousie was a staunch character and association with the Sattras,
opponent of vernacular education. this dance style has been aptly named
2. Hunter Commission was in favour Sattriya.
of the state sponsored vernacular Source:
education.
http://ccrtindia.gov.in/sattriya.php
Select the correct answer using the
codes given below:
89. With reference to the Surat Split,
(a) 1 only (b) 2 only
consider the following statements:
(c) Both 1 and 2 (d) Neither 1 nor 2
1. Tilak was the president of the
Explanation: Surat session of the Indian
Statement 1 is incorrect: In a famous National Congress.
minute, Lord Dalhousie expressed strong 2. Lord Minto was the viceroy of
opinion in favour of vernacular education. India during the split.
Statement 2 is correct: The Hunter Which of the statements given above
Education Commission held that State is/are correct?
should make special efforts for extension
(a) 1 only
and improvement of vernacular education.
Mass education was to be seen as (b) 2 only
instructing masses through vernaculars. (c) Both 1 and 2
Source: A Brief History of India; Chapter: (d) Neither 1 nor 2
Chapter 30: Development of Explanation:
Education; Page: 641 Statement 1 is incorrect: The Extremists
wanted the 1907 session to be held in
88. Identify the classical dance form using Nagpur (Central Provinces) with Tilak or
37

the features given below: Lajpat Rai as the president along with a
reiteration of the swadeshi, boycott and
Page

DELHI: VIJAY NAGAR 9717380832 & OLDRAJENDER NAGAR 9811293743 | JAIPUR: 8290800441
BENGALURU: KORMANGALA 7619166663 & CHANDRA LAYOUT 7619136663 | BHOPAL: 7509975361
PATNA: 7463950774 | INDORE: 7314977441 | RANCHI: 7463950774 | www.ksgindia.com
TEST - 1122 (EXPLANATION & SOURCE)
national education resolutions. The Source: A Brief History of Modern India;
Moderates wanted the session at Surat in Chapter 32: The Movement of the
order to exclude Tilak from the presidency, Working Class; Page: 660
since a leader from the host province could
not be session president (Surat being in
Tilak’s home province of Bombay). Instead,
they wanted Rashbehari Ghosh as the
president and sought to drop the resolutions 91. Justice Movement was started by:
on swadeshi, boycott and national (a) N. Kumaran Asan and T.K.
education. But LajpatRai stepped down and Madhavan
Ghosh became the President. (b) T.K. Madhavan, T.M. Nair and P.
Statement 2 is correct: Both sides adopted Tyagaraja
rigid positions, leaving no room for (c) K.Kelappan, C.N. Mudaliar and
compromise. The split became inevitable, T.K. Madhavan
and the Congress was now dominated by the (d) C.N. Mudaliar, T.M. Nair and P.
Moderates who lost no time in reiterating Tyagaraja
Congress’ commitment to the goal of self-
Explanation:
government within the British Empire and to
the use of constitutional methods only to Option (d) is correct
achieve this goal. Justice movement in Madras Presidency was
Lord Minto was the viceroy of India in 1907. started by C.N. Mudaliar, T.M. Nair and P.
Tyagaraja to secure jobs and representation
Source: Modern History/Spectrum/ pg.
for the non-brahmins in the legislature. In
305-306
1917, Madras Presidency Association was
formed which demanded separate
90. Which of the following is/are the representation for the lower castes in the
features of the Trade Union Act, 1926? legislature
1. Recognition of the trade unions as Source: A Brief History of Modern India,
legal entities. Page 249
2. Secured immunity, both civil and
criminal, for trade unions from 92. In context of the Lomus Rishi cave
prosecution for legitimate consider the following statements:
activities
1. It is a rock-cut cave at Barabar
Select the correct answer using the hills near Gaya in Bihar
codes given below:
2. The facade of the cave is decorated
(a) 1 only with the semicircular chaitya arch
(b) 2 only as the entrance.
(c) Both 1 and 2 3. The cave was patronised by
(d) Neither 1 nor 2 Bindusara for the Ajivika sect.
Explanation: Which of the statements given above
Option (c) is correct: The Trade Union Act, is/are correct?
1926 (a) 1 and 2 only
● recognised trade unions as legal (b) 2 and 3 only
associations; (c) 1 and 3 only
● laid down conditions for registration (d) 1, 2 and 3
and regulation of trade union activities; Explanation:
● secured immunity, both civil and Option (a) is correct:
criminal, for trade unions from
The rock-cut cave carved at Barabar hills
prosecution for legitimate activities, but
near Gaya in Bihar is known as the Lomas
38

put some restrictions on their political


Rishi cave. The facade of the cave is
activities.
decorated with the semicircular chaitya arch
Page

DELHI: VIJAY NAGAR 9717380832 & OLDRAJENDER NAGAR 9811293743 | JAIPUR: 8290800441
BENGALURU: KORMANGALA 7619166663 & CHANDRA LAYOUT 7619136663 | BHOPAL: 7509975361
PATNA: 7463950774 | INDORE: 7314977441 | RANCHI: 7463950774 | www.ksgindia.com
TEST - 1122 (EXPLANATION & SOURCE)
as the entrance. The elephant frieze carved adorned with elaborate head-dresses, and
in high relief on the chaitya arch shows sometimes painted with masks also.
considerable movement. The interior hall of Elephants, bison, tiger, boar, deer, antelope,
this cave is rectangular with a circular leopard, panther, rhinoceros, fish, frog,
chamber at the back. The entrance is located lizard, squirrel and at times birds are also
on the side wall of the hall. The cave was depicted. TheMesolithic artists loved to paint
patronised by Ashoka for the Ajivika sect. animals. In some pictures, animals are
The Lomas Rishi cave is an isolated example chasing men. In others they are being
of this period. But many Buddhist caves of chased and hunted by men. Some of the
the subsequent periods were excavated in animal paintings, especially in the hunting
eastern and western India. scenes, show a fear of animals, but many
Source: Ch-3, Arts of MauryaPeriod,An others show a feeling of tenderness and love
Introduction toIndian Art Part-I, Class-XI. for them. There are also a few engravings
representing mainly animals.
93. Consider the following statements: Though animals were painted in a
naturalistic style, humans were depicted
1. The paintings of the Upper
only in a stylistic manner. Womenare
Palaeolithic phase were linear
painted both in the nude and clothed. The
representations in green and dark
young and the old equally find place in these
red colour.
paintings. Children are painted running,
2. In Mesolithic paintings, animals jumping and playing. Community dances
were painted in a naturalistic provide a common theme. There are
style, while humans were depicted paintings of people gathering fruit or honey
only in a stylistic manner. from trees, and of women grinding and
Which of the statements given above preparing food. Some of the pictures of men,
is/are correct? women and children seem to depict a sort of
(a) 1 only family life. In many of the rock-shelters we
(b) 2 only find handprints, fist prints, and dots made
(c) Both 1 and 2 by the fingertips.
(d) Neither 1 nor 2 Source: An Introduction to Indian Art,
NCERT XI, Chapter 1, Page- 4
Explanation:
Both the statements are correct-The
paintings of the UpperPalaeolithic phase are 94. Consider the following pairs:
linear representations, in green and dark Literary Sources Writers
red, of huge animal figures, such as bisons, 1. Arthasastra Kautilya
elephants, tigers, rhinos and boars besides 2. Indica Megasthenes
stick-like human figures. A few are wash 3. Mudrarakshasa Visakadatta
paintings but mostly they are filled with
Which of the pairs given above are
geometric patterns. The green paintings are
correctly matched?
of dancers and the red ones of hunters.
(a) 1 and 2 only
The largest number of paintings belong to
Period II that covers the Mesolithic (b) 1 and 3 only
paintings. During this period the themes are (c) 2 and 3 only
multiple but the paintings are smaller in (d) 1, 2 and 3
size.Hunting scenes predominate. The Explanation:
hunting scenes depict people hunting in Pair 1 is correctly matched: Arthasastra
groups, armed with barbed spears,pointed book in Sanskrit was written by Kautilya, a
sticks, arrows and bows. In some paintings contemporary of Chandragupta Maurya.
these primitive men are shown with traps Kautilya was also called ‘Indian Machiavelli’.
and snares probably to catch animals. The The manuscript of Arthasastra was first
39

hunters are shown wearing simple clothes discovered by R. ShamaSastri in 1904. The
and ornaments.Sometimes, men have been Arthasastra contains 15 books and 180
Page

DELHI: VIJAY NAGAR 9717380832 & OLDRAJENDER NAGAR 9811293743 | JAIPUR: 8290800441
BENGALURU: KORMANGALA 7619166663 & CHANDRA LAYOUT 7619136663 | BHOPAL: 7509975361
PATNA: 7463950774 | INDORE: 7314977441 | RANCHI: 7463950774 | www.ksgindia.com
TEST - 1122 (EXPLANATION & SOURCE)
chapters but it can be divided into three as well as against powerful Turkish nobles,
parts: the first deals with the king and his and could rule only for three years.
council and the departments of government; Though brief, her rule had a number of
the second with civil and criminal law; and interesting features. It marked the beginning
the third with diplomacy and war. It is the of a struggle for power between the
most important literary source for the monarchy and the Turkish chiefs,
history of the Mauryas. sometimes called ‘the forty’ or the
Pair 2 is correctly matched: Megasthenes chahalgani. Iltutmish had shown great
was the Greek ambassador in the court of deference to these Turkish chiefs. After his
Chandragupta Maurya. His book Indica has death, these chiefs, drunk with power and
survived only in fragments. Yet, his account arrogance, wanted to install on the throne a
gives details about the Mauryan puppet whom they could control.
administration, particularly the Source-History of Medieval India, Satish
administration of the capital city of Chandra, Chapter 5, Page- 94
Pataliputra and also the military
organization. His picture on contemporary
96. Consider the following statements:
social life is notable. Certain unbelievable
information provided by him has to be 1. While Ashoka believed in a policy
treated with caution. of peace and non-aggression,
Samudragupta was delighted in
Pair 3 is correctly matched:
violence and conquest.
The Mudrarakshasa written by Visakadatta
2. The inscription by Harisena about
is a drama in Sanskrit. Although written
Samudragupta was engraved at
during the Gupta period, it describes how
Allahabad on the same pillar that
Chandragupta with the assistance of
carries the inscriptions of the
Kautilya overthrew the Nandas. It also gives
peace-loving Ashoka.
a picture on the socio-economic condition
under the Mauryas. Which of the statements given above
is/are correct?
Source: class11 History/TN Board/Pg. 59-
60 (a) 1 only
(b) 2 only
95. With reference to Delhi Sultanate, the (c) Both 1 and 2
term “chahalgani” refers to? (d) Neither 1 nor 2
(a) Turkish Chiefs Explanation:
(b) Polo game Both the statements are correct-The
(c) Hunting game Gupta kingdom was enlarged enormously by
Chandragupta’s son and successor
(d) Group of Ulemas
Samudragupta (AD 335–80). He was the
Explanation: Option (a) is correct opposite of Ashoka. Ashoka believed in a
During his last year, Iltutmish was worried policy of peace and non-aggression, but
over the problem of succession. He Samudragupta delighted in violence and
considered none of his surviving sons to be conquest. His court poet Harishena wrote a
worthy of the throne. After anxious glowing account of the military exploits of his
consideration, he finally decided to nominate patron, and, in a long inscription, the poet
his daughter, Raziya, to the throne, and enumerate the peoples and countries that
induced the nobles and the theologians were conquered by Samudragupta. The
(ulama) to agree to the nomination. Although inscription is engraved at Allahabad on the
women had ruled as queens, both in ancient same pillar that carries the inscriptions of
Iran and Egypt, and had acted as regents the peace-loving Ashoka.
during the minority rule of princes, the Source-India’s Ancient Past,R.S.Sharma,
nomination of a woman in preference to sons Chapter-24,Page 275
40

was a novel step. In order to assert her claim,


Raziya had to contend against her brothers
Page

DELHI: VIJAY NAGAR 9717380832 & OLDRAJENDER NAGAR 9811293743 | JAIPUR: 8290800441
BENGALURU: KORMANGALA 7619166663 & CHANDRA LAYOUT 7619136663 | BHOPAL: 7509975361
PATNA: 7463950774 | INDORE: 7314977441 | RANCHI: 7463950774 | www.ksgindia.com
TEST - 1122 (EXPLANATION & SOURCE)
97. Which among the following are Source: People’s Resistance Against British
correctly matched? Before 1857, page No. 171-174
Tribal Uprising Area where started
98. With reference to Magadha Kingdom of
North India, consider the following
1. Pahariyas Raj Mahal Hills
statements:
1. Rajagriha was the capital of
2. Chuar Singhbum Magadha kingdom.
2. Bimbisara was a contemporary of
3. Ho and Munda Midnapore both Vardhamana Mahavira and
Gautama Buddha.
3. Ajatasatru was instrumental in
4. Bhil Revolt Western Ghats
convening the fourth Buddhist
Council at Rajagriha
Select the correct answer using the
code given below: Which of the statements given above
is/are correct?
(a) 1 and 4 only
(a) 1 only
(b) 2 and 3 only
(b) 1 and 2 only
(c) 1 and 3 only
(c) 2 and 3 only
(d) 2 and 4 only
(d) 1, 2 and 3
Explanation Option (a) is correct
Explanation:
Pahariyas’ Rebellion
Statement 1 is correct: Of all the kingdoms
The British expansion on their territory led
of north India, Magadha emerged powerful
to an uprising by the martial Pahariyas of
and prosperous. It became the nerve centre
the Raj Mahal Hills in 1778. The British were
of political activity in north India. Magadha
forced to usher in peace by declaring their
was endowed by nature with certain
territory as damni-kol area.
geographical and strategic advantages.
Chuar Uprising These made her to rise to imperial greatness.
Famine, enhanced land revenue demands Her strategic position between the upper and
and economic distress goaded the Chuar lower part of the Gangetic valley was a great
aboriginal tribesmen of the Jungle Mahal of advantage. It had a fertile soil. The iron ores
Midnapore district and also of the Bankura in the hills near Rajgir and copper and iron
district (in Bengal) to take up arms. These deposits near Gaya added to its natural
tribes people were basically farmers and assets. Her location at the centre of the
hunters. highways of trade of those days contributed
Ho and Munda Uprisings (1820-1837) to her wealth. Rajagriha was the capital of
The Raja of Parahat organised his Ho tribals Magadha. During the reign of Bimbisara and
to revolt against the occupation of Ajatasatru, the prosperity of Magadha
Singhbhum (now in Jharkhand). The revolt reached its zenith.
continued till 1827 when the Ho tribals were Statement 2 is correct:Bimbisara belonged
forced to submit. to the Haryanka dynasty. He consolidated
Bhil Revolts his position by matrimonial alliances. His
The Bhils who lived in the Western Ghats first matrimonial alliance was with the ruling
controlled the mountain passes between the family of Kosala. He married Kosaladevi,
north and the Deccan. They revolted against sister of Prasenajit. He was given the Kasi
Company rule in 1817-19, as they had to region as dowry which yielded large revenue.
face famine, economic distress and Bimbisara married Chellana, a princess of
misgovernment. The British used both force the Licchavi family of Vaisali. This
and conciliatory efforts to control the matrimonial alliance secured for him the
41

uprising. safety of the northern frontier. Moreover, it


facilitated the expansion of Magadha
Page

DELHI: VIJAY NAGAR 9717380832 & OLDRAJENDER NAGAR 9811293743 | JAIPUR: 8290800441
BENGALURU: KORMANGALA 7619166663 & CHANDRA LAYOUT 7619136663 | BHOPAL: 7509975361
PATNA: 7463950774 | INDORE: 7314977441 | RANCHI: 7463950774 | www.ksgindia.com
TEST - 1122 (EXPLANATION & SOURCE)
northwards to the borders of Nepal. He also (a) 1 only
married Khema of the royal house of Madra (b) 2 only
in central Punjab. Bimbisara also undertook (c) Both 1 and 2
many expeditions and added more territories
(d) Neither 1 nor 2
to his empire. He defeated Brahmadatta of
Anga and annexed that kingdom. He Explanation:
maintained friendly relations with Avanti. He Statement 1 is correct and Statement 2 is
had also efficiently reorganized the incorrect- Although the greatest amount of
administration of his kingdom. Bimbisara literature and scientific works produced by
was a contemporary of both the Muslims was in Arabic which was the
VardhamanaMahavira and Gautama language of the Prophet and was used as the
Buddha. However, both religions claim him language of literature and science from
as their supporter and devotee. He seems to Spain to Baghdad, theTurks who came to
have made numerous gifts to the Buddhist India were deeply influenced by the Persian
Sangha. language which had become the literary and
Statement 3 is incorrect: administrative language of Central Asiafrom
the tenth century onwards. In India, the use
The reign of Ajatasatru was remarkable for
of Arabic remained largely confined to a
his military conquests. He fought against
narrow circle of Islamic scholars and
Kosala and Vaisali. His won a great success
philosophers, most of the original literature
against a formidable confederacy led by the
on the subject being written in Arabic. A few
Lichchavis of Vaisali. This had increased his
works on science and astronomy were also
power and prestige. This war lasted for about
translated from Arabic. In course of
sixteen years. It was at this time that
time,digests of the Islamic law were prepared
Ajatasatrurealised the strategic importance
in Persian with the help of Indianscholars.
of the small village, Pataligrama (future
The most well-known of these were prepared
Pataliputra). He fortified it to serve as a
in the reign of FiruzTughlaq. But Arabic
convenient base of operations against
digests continued to be prepared, the most
Vaisali. Buddhists and Jains both claim that
famous of these being the Fatawa-i-Alamgiri,
Ajatasatru was a follower of their religion.
or the Digest of Laws prepared by a group
But it is generally believed that in the
of jurists in the reign of Aurangzeb.
beginning he was a follower of Jainism and
subsequently embraced Buddhism. He is Sultan Zain-ul-Abidin of Kashmir had the
said to have met Gautama Buddha. This famous historical work Rajatarangini and
scene is also depicted in the sculptures of the Mahabharata translated into Persian.
Barhut. According to the Mahavamsa, he At his instance, Sanskrit works on medicine
constructed several chaityas and viharas. and music were also translated intoPersian.
He was also instrumental in convening Recent research shows that some works on
the First Buddhist Council at Rajagriha mathematics, astonomy and medicine were
soon after the death of the Buddha. translated into Sanskrit during the period.
Source: class11 History/TN Board/Pg. 49- Source:History of Medieval India, Satish
50 Chandra, Chapter-11, Page- 211-213

99. Consider the following statements: 100. Consider the following statements
about Indo-Islamic architecture in
1. Sultan Zain-ul-Abidin of Kashmir
India:
had the Rajatarangini and the
Mahabharata translated into 1. The Alai-Darwaza was built by a
Persian. Mughal king.
2. The Fatawa-i-Alamgiri, or the 2. Red sandstone is used in its
Digest of Laws was prepared in the construction.
reign of Shah Jahan. Which of the statements given above
42

Which of the statements given above is/are correct?


is/are correct? (a) 1 only
Page

DELHI: VIJAY NAGAR 9717380832 & OLDRAJENDER NAGAR 9811293743 | JAIPUR: 8290800441
BENGALURU: KORMANGALA 7619166663 & CHANDRA LAYOUT 7619136663 | BHOPAL: 7509975361
PATNA: 7463950774 | INDORE: 7314977441 | RANCHI: 7463950774 | www.ksgindia.com
TEST - 1122 (EXPLANATION & SOURCE)
(b) 2 only arch in the form of a pointed horseshoe,
(c) Both 1 and 2 broad dome, recessed arches under the
(d) Neither 1 nor 2 squinch, perforated windows, inscriptional
bands and use of red sandstone relieved by
Explanation:
marble are features characteristic of Khilji
Statement 1 is incorrect: The Alai- architecture.
Darwaza was, built by AllaudinKhilji by
Source:
enlarging the Quwwat-ul--Islam Mosque's
enclosures of colonnades and providing http://ccrtindia.gov.in/indoislamicarchitect
them two gateways. ure.php
Statement 2 is correct: In this and other
buildings constructed by the Khiljis, the true

43
Page

DELHI: VIJAY NAGAR 9717380832 & OLDRAJENDER NAGAR 9811293743 | JAIPUR: 8290800441
BENGALURU: KORMANGALA 7619166663 & CHANDRA LAYOUT 7619136663 | BHOPAL: 7509975361
PATNA: 7463950774 | INDORE: 7314977441 | RANCHI: 7463950774 | www.ksgindia.com

You might also like